NURS 220 Final Video Questions, Evolve quiz questions, and prep work quiz questions

¡Supera tus tareas y exámenes ahora con Quizwiz!

A nursing assistive personnel asks for help to transfer a patient who is 125 Ibs (56.8 kg) from the bed to a wheelchair. The patient is unable to help. What is the nurse's best response? "The two of us can lift the patient easily." "Call the lift team for additional assistance." "As long as we use proper body mechanics, no one will get hurt." "The patient only weighs 125 Ibs, You don't need my assistance. "

"Call the lift team for additional assistance."

You are checking the pH of gastric aspirate for a patient that is receiving enteral feeding. What should be the result of the pH? -1.3 -2.5 -8.0 -5.0 or higher

-5.0 of higher

What is the proper way to measure the length of NGT to be inserted? -from the inner canthus of the eye to the nose and then to the xiphoid process -From the tip of the nose to earlobe and then to the xiphoid process -From the xiphoid process to the earlobe to the tip of the nose -from the earlobe to the tip of the nose and then to the xiphoid process

-From the tip of the nose to the earlobe and then to the xiphoid process

Which nursing intervention will best promote respiratory health in an immobile postoperative patient? -Isometric exercises -Sequential compression device use -incentive spirometry -low-dose heparin therapy

-Incentive spirometry

Skin barrier or (stoma paste) should be used for what purpose? -help prevent odor -Protect the skin from effluent -Allow for the pouch to stay on more than expected -Help with charting input and output

-Protect the skin from effluent

In what position should a patient be when receiving an enema? -fowlers -reverse trendelenburg -Sims, laying on the left with right knee flexed -supine

-Sims, laying on the left with right knee flexed

In order to maintain proper body alignment and posture you must: (select all that apply) -Bend at the hips and lift using the muscles in your back for added support -Widen the base of support by separating the feet to a comfortable distance -Ensure the bed is at its lowest to prevent the risk of falls -Increase balance by bringing the center of gravity closer to the base of support

-Widen the base of support by separating the feet to a comfortable distance -Increase balance by bringing the center of gravity closer to the base of support

Which of these statements are true regarding disinfection and cleaning? (select all that apply) 1. Proper cleaning requires mechanical removal of all soil from an object or area 2. General environmental cleaning is an example of medical asepsis 3. When cleaning a wound, wipe around the wound edge first then clean inward toward the center of the wound 4. Cleaning in a direction from the least to the most contaminated area helps reduce infections 5. Disinfecting and sterilizing medical devices and equipment involve the same procedures

1, 2, 4

Put the following steps for removal of PPE after leaving an isolation room in order: 1. Remove gloves 2. Perform hand hygiene 3. Remove eyewear or goggles 4. Untie top and then bottom mask strings and remove from face 5. Untie waist and neck strings of gown. Remove gown, rolling it onto self without touching the contaminated side

1, 3, 5, 4, 2

Place the following options in the order in which elastic stockings should be applied 1. Identify patient using two identifiers. 2. Use tape measure to measure patient's leg to determine proper stocking size 3. Slide the remainder of the stocking over the patient's heel and up the leg 4. Assess the condition of the patient's skin and circulation of the legs. 5.Turn the stocking inside out until heel is reached 6. Place toes into foot of the stocking. 7. Smooth any creases or wrinkles.

1, 4, 2, 5, 6, 3, 7

Match the definition with the term: 1. Isometric exercises 2. Friction 3.Resistive isometric exercises 4. Shear A. those in which an individual contracts the muscle while pushing against a stationary object or resisting the movement of an object B. Tightening or tensing muscles without moving body parts C. A force that occurs in a direction to oppose movement D. the force exerted against the skin while the skin remains stationary and the bony structures move

1. B 2. C 3. A 4. D

A health care provider ordered enalapril (Vasotec) 2 mg IV push for a patient with hypertension. The pharmacy sent vials marked 1.25 mg enalapril/mL. How many mL does the nurse administer?

1.6mL

Match the relationship 1.Calcium resorption 2.Endocrine system 3.Tissue catabolism 4.GI distrubances such as decreased appetite and slowing of peristalsis 5.Virchow's triad A. Weight loss, decreased muscle mass, and weakness B. Constipation or pseudodiarrhea. Dehydration, ceased absorption, and fluid and electrolytes C. Helps regulate through maintenance of sodium, potassium, water, and acid-base balance. D. Pathological fractures E. damage to the vessel wall, alterations in blood flow, and alterations in blood constituents

1.D 2.C. 3.A. 4.B 5.E

The nurse is administering an intravenous (IV) push medication to a patient who has a compatible IV fluid running through intravenous tubing. Place the following steps in the appropriate order. 1. Release tubing and inject medication within amount of time recommended by agency policy, pharmacist, or medication reference manual. Use watch to time administration. 2. Select injection port of intravenous (IV) tubing closest to patient. Whenever possible, injection port should accept a needleless syringe. Use IV filter if required by medication reference or agency policy. 3. After injecting medication, release tubing, withdraw syringe, and recheck fluid infusion rate. 4. Connect syringe to port of intravenous (IV) line. Insert needleless tip or small-gauge needle of syringe containing prepared drug through center of injection port. 5. Clean injection port with antiseptic swab. Allow to dry. 6. Occlude intravenous (IV) line by pinching tubing just above injection port. Pull back gently on syringe plunger to aspirate blood return.

2, 5, 4, 6, 1, 3

A patient's surgical wound has become swollen, red, and tender. The nurse notes that the patient has a new fever, purulent wound drainage, and leukocytosis. Which interventions would be appropriate and in what order? 1. Notify the health care provider of the patient's status 2. Reassure the patient and recheck the wound later 3. Support the patient's fluid and nutritional needs 4. Use aseptic technique to change the dressing

4, 2, 1, 3

A child is taking albuterol through a pressurized metered-dose inhaler (pMDI) that contains a total of 64 puffs. The dose is 2 puffs every 6 hours. How many days will the pMDI last? ___________ days

8 days

. A patient with type 2 diabetes mellitus tells the nurse that he has been testing his own blood glucose level six times per day for the past 3 years. What is the most appropriate action for the nurse to take? A. Observe the patient's testing technique for accuracy B. Advise the patient that he is not permitted to perform his own blood glucose testing C. Check with the patient's health care provider concerning the patient's self-testing D. Explain to the patient that a nurse must complete blood glucose testing

A

1. A patient has been on bed rest for over 4 days. On assessment, the nurse identifies the following as a sign associated with immobility: a. Decreased peristalsis b. Decreased heart rate c. Increased blood pressure d. Increased urinary output

A

1. The nurse is ambulating a patient with a gait belt when he says he feels sick to his stomach. What would the nurse do? a. Return the patient to the bed or chair (whichever is closer) b. Encourage the patient to complete the distance of ambulation c. Help him to the restroom d. Ease him to the floor

A

100. Which nursing action is appropriate when feeding gastric residual is 50 mL? a. Return it to the stomach via the feeding tube. b. Dispose of the residual contents down the commode. c. Discard the stomach contents as a liquid biohazard. d. Return half of the volume to the stomach, and discard the rest

A

106. The nurse is caring for a patient who is recovering from a left partial lobectomy. Which action would be most helpful in reexpanding the affected lung? a. Placing the patient in a right side-lying position b. Encouraging the patient to deep breathe and cough every hour c. Regularly assessing the patient's ability to breathe comfortably d. Providing medication to manage postoperative pain of greater than 3 on a 0-to-10 scale

A

14. To which position would the nurse assist the patient who is experiencing difficulty with breathing? a. Fowler's position b. 30-degree lateral position c. Sim's position d. Prone position

A

16. A patient is experiencing some problems with joint stability. The doctor has prescribed crutches for the patient to use while still being allowed to bear weight on both legs. Which of the following gaits should the patient be taught to use? a. Four-point b. Three-point c. Two-point d. Swing-through

A

18. The nurse recognizes that the older adult's progressive loss of total bone mass and tendency to take smaller steps with feet kept closer together will most likely: a. Increase the patient's risk for falls and injuries. b. Result in less stress on the patient's joints. c. Decrease the amount of work required for patient movement. d. Allow for mobility in spite of the aging effects on the patient's joints.

A

2. A nurse is caring for an older adult who has had a fractured hip repaired. In the first few postoperative days, which of the following nursing measures will best facilitate the resumption of activities of daily living for this patient? a. Encouraging use of an overhead trapeze for positioning and transfer. b. Frequent family visits c. Assisting the patient to a wheelchair once per day d. Ensuring that there is an order for physical therapy

A

21. When turning a patient to place a slide board, where do the assistants stand? a. At the side of the bed to which the patient will be turned b. At the side of the bed from which the patient will be turned c. At the head and foot of the bed d. At the foot of the bed only

A

23. The nurse and his or her assistants are using a slide board to move a patient from the bed to a stretcher. The nurse, standing alone on the side of the bed opposite the stretcher, will perform which action during the move? a. Hold the slide board stationary b. Pull the draw sheet c. Hold the patient's head stationary d. Lock the brakes on the stretcher

A

32. An 86 year old woman is admitted to the unit with chills and a fever of 104 degrees F. What physiological process explains why she is at risk for dyspnea? a. Fever increases metabolic demands requiring increased oxygen need. b. Blood glucose stores are depleted and the cells do not have energy to use oxygen. c. Carbon dioxide production increases due to hyperventilation. d. Carbon dioxide production decreases due to hypoventilation

A

32. When preparing to apply elastic stockings, why does the nurse assess for skin discoloration? a. To identify the potential risk of DVT b. To identify improper patient positioning c. To select the proper stocking size d. To determine whether a sequential compression device is needed

A

37. The nurse goes to assess a new patient and finds him short of breath with a rate of 32 and lying supine in bed. What is the priority nursing action? a. Raise the head of the bed to 60 degrees or higher. b. Get his oxygen saturation with a pulse oximeter. c. Take his blood pressure and respiratory rate. d. Notify the health care provider of his shortness of breath.

A

38. Two hours after surgery, the nurse assesses a patient who had a chest tube inserted during surgery. There is 200 ml of dark red drainage in the chest tube at this time. What is the appropriate action for the nurse to perform? a. Record the amount and continue to monitor drainage. b. Notify the physician. c. Strip the chest tube starting at the chest d. Increase the suction by 10 mm Hg

A

38. Why would the nurse as a physical therapist to perform passive ROM exercises for a patient with lower extremity injuries sustained in a motor vehicle crash? a. The patient is an older adult or has a chronic condition b. The patient is reluctant to perform the exercises because he is worried about reinjury c. The patient has orthopedic trauma d. The patient has pain exacerbated by exercise

A

4. The nurse has applied a gait belt to a postoperative patient to facilitate ambulation. Within a few feet of the bed, the patient begins to complain of dizziness and leans heavily on the nurse. What would be the nurse's initial response? a. Slowly lower the patient to the floor b. Attempt to sit the patient down on a chair just a few steps away c. Try to hold the patient up until the dizziness passes d. Call for assistance in a loud but calm voice

A

4. The nurse is caring for a patient whose calcium intake must increase because of high risk factors for osteoporosis. Which of the following menus should the nurse recommend? a. Cream of broccoli soup with whole wheat crackers, cheese, and tapioca for dessert b. Hot dog on whole wheat bun with a side salad and an apple for dessert c. Low-fat turkey chili with sour cream with a side salad and fresh pears for dessert d. Turkey salad on toast with tomato and lettuce and honey bun for dessert

A

42. The nurse is preparing to administer an enema. How can the nurse best facilitate insertion of the rectal tube? a. Lubricate the first 6.5 to 7.5 cm (2.5 to 3 inches) of the tip of the tube b. Place the patient in a side-lying position with the right knee flexed. c. Flush the tube with the solution. d. Hold the tube in the rectum until all of the fluid has been instilled.

A

45. The nurse is caring for a patient with dysphagia and is feeding her a pureed chicken diet when she begins to choke. What is the priority nursing intervention? a. Suction her mouth and throat b. Turn her on their side c. Put on oxygen at 2-L nasal cannula d. Stop feeding her and place on NPO

A

46. A patient who is receiving parenteral nutrition (PN) through a central venous catheter (CVC) has an air embolus. What would the nurse do first? a. Have the patient perform a Valsalva procedure b. Clamp the intravenous (IV) tubing to prevent more air from entering the line c. Have the patient take a deep breath and hold it d. Notify the health care provider immediately

A

54. A male patient on bed rest is permitted to stand to use the urinal. Which action would the nurse take to ensure his safety before helping him to a standing position? a. Determine his risk for orthostatic hypotension b. Assess his genitals for signs of impaired skin integrity c. Ask him to demonstrate proper use of a urinal d. Instruct him to use the call light when he is finished

A

6. In which position will the nurse place the patient to move him or her up in bed? a. Supine with the head of the bed flat b. Sitting in the bed c. Supine with the head of the bed at a 30-degree angle d. Prone with the head of the bed flat

A

60. A patient has not had a bowel movement for 4 days. Now she has nausea and severe cramping throughout her abdomen. On the basis of these findings, what should the nurse suspect? a. An intestinal obstruction b. Irritation of the intestinal mucosa c. Gastroenteritis d. A fecal impaction

A

61. During the administration of a warm tap-water enema, the patient complains of cramping abdominal pain that he rates 6 out of 10. What is the first thing the nurse should do? a. Stop the instillation b. Ask the patient to take deep breaths to decrease the pain c. Add soapsuds to the enema d. Tell the patient to bear down as he would when having a bowel movement

A

67. After administering a rectal suppository for constipation, the nurse will monitor for all of the following responses except which one? a. Low platelet count b. Rectal pain c. Bradycardia d. Evacuation of stool

A

70. The nurse should question a provider's order to insert a suppository into the rectum of a patient with which condition? a. Watery diarrhea b. Rectal inflammation c. External hemorrhoids d. Internal hemorrhoids

A

77. What might the nurse do to reduce the patient's discomfort before inserting a nasogastric tube? a. Examine each naris for patency and skin breakdown. b. Place the patient in the high-Fowler's position. c. Anesthetize the throat. d. Have the patient take a few sips of water.

A

78. Which patient does not have a medical condition that contraindicates placement of a nasogastric tube? a. A 28-year-old patient who fractured a femur after heavy drinking. b. A 73-year-old patient who is on anticoagulation therapy. c. A 54-year-old patient who broke a cheekbone in a fall. d. A 67-year-old patient with a history of unexplained nosebleeds

A

79. Which intervention might the nurse delegate to nursing assistive personnel (NAP) when inserting a nasogastric tube? a. Positioning the patient in a high-Fowler's position b. Assessing the patient's abdomen for bowel sounds c. Determining any history of unexplained nosebleeds d. Educating the patient about the need for the intervention

A

88. Why is it important for the nurse to set the correct flow rate for a patient to whom oxygen is prescribed? a. To provide the correct amount of oxygen to the patient b. To ensure the therapeutic effects of oxygen therapy c. To prevent any adverse reaction to the prescribed oxygen therapy d. To minimize the risk of combustion during oxygen delivery

A

89. What can the nurse do to evaluate a patient's response to continuous oxygen therapy delivered at 4 L/min by nasal cannula? a. Regularly measure and trend the patient's pulse oximetry (SpO2) values. b. Evaluate venous blood levels every morning. c. Monitor the patient's arterial blood gas (ABG) levels hourly. d. Assess the patient for compliance with the prescribed therapy.

A

93. What is the nurse's initial action when preparing to change a patient's colostomy pouching system? a. Applying clean gloves b. Draping the patient appropriately c. Emptying the colostomy d. Assessing the surrounding skin for signs of irritation.

A

99. Why does the nurse elevate the head of the bed to 30 degrees for a patient receiving an intermittent tube feeding? a. Elevating the head of the bed reduces the risk for aspiration. b. Proper elevation of the head of the bed promotes the patient's digestion. c. Acid reflux is reduced when the head of the bed is elevated at least 30 degrees. d. Nutrients are absorbed more efficiently when the head of the bed is elevated.

A

A 55-year-old female patient was in a motor vehicle accident and is admitted to a surgical unit after repair of a fractured left arm and left leg. She also has a laceration on her forehead. An intravenous (IV) line is infusing in the right antecubital fossa, and pneumatic compression stockings are on the right lower leg. She is receiving oxygen via a simple face mask. Which sites do you instruct the nursing assistant to use for obtaining the patient's blood pressure and temperature? A. Right antecubital and tympanic membrane B. Right popliteal and rectal C. Left antecubital and oral D. Left popliteal and temporal artery

A

A male patient receiving perineal care tells the nurse "It has started to hurt a little down there." What is the nurse's best response? A. "When did you start experiencing the pain?" B. "Rate the pain on a scale of 1 to 10." C. "I'll assess your perineal area for the possible cause of the pain." D. "Would you like some pain medication before I continue with your care?"

A

A new nurse complains to her preceptor that she has no time for therapeutic communication with her patients. Which of the following is the best strategy to help the nurse find more time for this communication? A. Include communication while performing tasks such as changing dressings and checking vital signs. B. Ask the patient if you can talk during the last few minutes of visiting hours. C. Ask Pastoral care to come back a little later in the day. D. Remind the nurse to complete all her tasks and then set up remaining time for communication.

A

A nurse has been gathering physical assessment data on a patient and is now listening to the patient's concerns. The nurse sets a goal of care that incorporates the patient's desire to make treatment decisions. This is an example of the nurse engaged in which phase of the nurse-patient relationship? A. Working phase B. Preinteraction phase C. Termination phase D. Orientation phase

A

A nurse is caring for an older adult who has had a fractured hip repaired. In the first few postoperative days, which of the following nursing measures will best facilitate the resumption of activities of daily living for this patient? A. Encouraging use of an overhead trapeze for positioning and transfer. B. Frequent family visits C. Assisting the patient to a wheelchair once per day D. Ensuring that there is an order for physical therapy

A

A nurse is checking a patient's intravenous line and, while doing so, notices how the patient bathes himself and then sits on the side of the bed independently to put on a new gown. This observation is an example of assessing: A. Patient's level of function B. Patient's willingness to perform self-care C. Patient's level of consciousness D. Patient's health management values

A

A nurse is listening to a student provide instruction to a patient who is having difficulty with activities needed to care for soft contact lenses. Which of the following statements by the nursing student might require some correction by the nurse? A. Use tap water to clean soft lenses. B. Follow recommendations of lens manufacturer when inserting the lenses. C. Keep lenses moist or wet when not worn. D. Use fresh solution daily when storing and disinfecting lenses

A

A nurse is preparing to help a patient administer a mucolytic agent using a metered-dose inhaler (MDI). What will the nurse do first in order to evaluate the medication's effectiveness? A. Assess the patient's respiratory status before administration B. Warn against overuse of the inhaler C. Discuss the side effects of the particular drug D. Verify the patient's identification according to agency policy

A

A nurse uses long firm, strokes distal to proximal while bathing a patient's legs because: A. It promotes venous circulation. B. It covers a larger area of the leg. C. It completes care in a timely fashion. D. It prevents blood clots in legs.

A

A patient has been newly admitted to a medicine unit with a history of diabetes and advanced heart failure. The nurse is assessing the patient's fall risks. Which of the following is the proper order of steps for the "Timed Get-up and Go Test" (TGUGT)? 1. Have patient rise from straight-back chair without using arms for support. 2. Begin timing. 3. Tell patient to walk 10 feet as quickly and safely as possible to a line you marked on the floor, turn around, walk back, and sit down. 4. Check time elapsed. 5. Look for unsteadiness in patient's gait. 6. Have patient return to chair and sit down without using arms for support. A. 3, 1, 2, 5, 6, 4 B. 2, 1, 3, 5, 6, 4 C. 1, 2, 3, 6, 5, 4 D. 1, 2, 3, 5, 6, 4

A

A patient has been on bed rest for over 4 days. On assessment, the nurse identifies the following as a sign associated with immobility: A. Decreased peristalsis B. Decreased heart rate C. Increased blood pressure D. Increased urinary output

A

A patient is experiencing some problems with joint stability. The doctor has prescribed crutches for the patient to use while still being allowed to bear weight on both legs. Which of the following gaits should the patient be taught to use? A. Four-point B. Three-point C. Two-point D. Swing-through

A

A patient is prescribed continuous oxygen saturation monitoring. The nurse would confirm that the alarms have been set to which limits? A. Low of 85% and a high of 100% B. Low of 80% and high of 10% C. Low of 75% and high of 90% D. Low of 82% and high of 95%

A

A patient with a malignant brain tumor requires oral care. The patient's level of consciousness has declined, with the patient only being able to respond to voice commands. What is the correct order for administration of oral care? 1. If patient is uncooperative or having difficulty keeping mouth open, insert an oral airway. 2. Raise bed, lower side rail, and position patient close to side of bed with head of bed raised up to 30 degrees. 3. Using a brush moistened with chlorhexidine paste, clean chewing and inner tooth surfaces first. 4. For patients without teeth, use a toothette moistened in chlorhexidine rinse to clean oral cavity. 5. Remove partial plate or dentures if present. 6. Gently brush tongue but avoid stimulating gag reflex. A. 2, 5, 1, 3, 6, 4 B. 5, 1, 2, 3, 6, 4 C. 2, 5, 3, 1, 6, 4 D. 2, 1, 5, 3, 4, 6

A

A pediatric nurse takes a medication to a 12-year-old female patient. The patient tells the nurse to take it away because she is not going to take it. What is the nurse's next action? A. Ask the patient's reason for refusal B. Consult with the patient's parents for advice C. Take the medication away and chart the patient's refusal D. Tell the patient that her health care provider knows what is best for her

A

After requesting a narcotic pain medication, the patient refuses it after the nurse prepares the injection. What is the nurse's best initial response? A. Ask the patient the reason for his refusal B. Notify the physician and ask for a different type of pain medication C. Have another registered nurse witness the proper discarding of the drug D. Explain to the patient the need to manage pain effectively

A

At 12 noon the emergency department nurse hears that an explosion has occurred in a local manufacturing plant. Which action does the nurse take first? A. Prepare for an influx of patients B. Contact the American Red Cross C. Determine how to resume normal operations D. Evacuate patients per the disaster plan

A

For which patient would the nurse most likely ask for a podiatrist consult for nail care? A. A middle-aged man with type 2 diabetes mellitus who feels tingling in his right foot. B. A middle-aged man with mobility impairment that has lasted several weeks after a fall from a ladder. C. An older adult woman with dementia who has broken her pelvis after falling on the kitchen floor D. A 12-year-old girl with a broken foot

A

How can the nurse best ensure the patient's' safety when preparing insulin for administration? A. Obtain the patient's current blood glucose level B. Clean the injection site with an antibacterial swab C. Apply clean gloves D. Wipe the rubber seal of the vial with alcohol

A

How can the nurse prevent negative pressure from building up in the vial when preparing an injection? A. Inject an amount of air into the vial equivalent to the volume of medication to be withdrawn B. Insert the needle through the center of the rubber seal C. Keep the tip of the needle below the level of fluid in the vial D. Tap the barrel of the syringe to dislodge air bubbles

A

Inadequate oxygenation to the body will cause the radial pulse to become: A. Tachycardic B. Bradycardic C. Irregular D. Bounding

A

The licensed practice nurse (LPN) provides you with the change-of-shift vital signs on four of your patients. Which patient do you need to assess first? A. 84-year-old man recently admitted with pneumonia, RR 28, SpO2 89% B. 54-year-old woman admitted after surgery for fractured arm, BP 160/86 mm Hg, HR 72 C. 63-year-old man with venous ulcers from diabetes, temperature 37.3° C (99.1° F), HR 84 D. 77-year-old woman with left mastectomy 2 days ago, RR 22, BP 148/62

A

The nurse has just helped a patient into the bathtub. Before leaving the bathroom, what would the nurse do to help ensure the patient's safety A. Show him how to use the call signal. B. Place an "Occupied" sign on the door C. Check the cleanliness of the room D. Remove unneeded supplies from the bathroom

A

The nurse has just measured a patient's blood pressure and is waiting 2 minutes to measure the pressure again. What is the purpose of taking two measurements? A. Minimize the effect of anxiety B. Distract the patient C. Listen the second and third Korotkoff sounds D. Confirm that the cuff was applied correctly

A

The nurse has provided a patient with a PRN oral analgesic that may be repeated as needed every 6 to 8 hours. What is the most appropriate follow-up action to ensure appropriate pain management? A. Reassess the patient's pain in 30 to 40 minutes B. Document the patient's request for pain medication C. Administer the pain medication again in 6 hours D. Include the patient's pain history in the end-of-shift nursing report

A

The nurse has washed a patient's arms. Which area should the nurse wash next? A. Hands B. Chest C. Abdomen D. Legs

A

The nurse is caring for a patient whose calcium intake must increase because of high risk factors for osteoporosis. Which of the following menus should the nurse recommend? A. Cream of broccoli soup with whole wheat crackers, cheese, and tapioca for dessert B. Hot dog on whole wheat bun with a side salad and an apple for dessert C. Low-fat turkey chili with sour cream with a side salad and fresh pears for dessert D. Turkey salad on toast with tomato and lettuce and honey bun for dessert

A

The nurse is directing NAP to make an occupied bed. What will the nurse say to minimize the risk of disease transmission to staff and patient during the bed change? A. "You'll need to apply Standard Precautions during this task." B. "Soiled linen should be rolled toward your uniform." C. "Soiled linen should be kept away from your uniform." D. "Keep the linen bag at the foot of the bed."

A

The nurse is planning to insert an oral airway into an unconscious patient before performing mouth care. In which direction is the airway initially inserted into the patient's mouth? A. Upside down, or with the curve facing up B. Right side up, and with the curve facing down C. With the curve angled toward the patient's left cheek D. With the curve angled toward the patient's right cheek

A

The nurse is preparing to apply an estrogen patch to a patient who will be discharged with a prescription for the medication. What would the nurse do to ensure that the patient is able to apply the medication patch? A. Determine the patient's physical ability to grasp the patch B. Assess the patient's skin for appropriate application sites. C. Assess the patient's understanding of the medication's purpose D. Determine the patient's ability to recognize the medication's possible side effects

A

The nurse is preparing to change the soiled linen of a patient's unoccupied bed. Which precaution minimizes the risk of transmitting microorganisms? A. Perform hand hygiene and apply clean gloves. B. Place fresh linen on a clean beside table or chair C. Put soiled linen in a pillow case before placing in a hamper. D. Roll soiled linen together with the dirty sides toward the center

A

The nurse is preparing to help a patient use a dry powder inhaler. What will the nurse do first in order to evaluate the medication's effectiveness? A. Assess the patient's respiratory status B. Warn the patient against overuse of the inhaler C. Discuss the side effects of the particular drug D. Verify the patient's identity according to agency policy

A

The nurse is preparing to make an occupied bed for a patient who is on aspiration precautions. What will the nurse do to ensure the safety of this patient during the bed change? A. Keep the head of the bed no lower than a 30-degree angle B. Fold a pillow in half and place it under the patient's head. C. Lower the bed to a flat position and place two pillows beneath the patient's head D. Ask another caregiver to hold the patient's head during the bed change

A

The nurse is reviewing placement of an unfitted bottom sheet with NAP assigned to make an unoccupied bed. What should the nurse include in this teaching? A. The lower hem of the sheet would lie seam down and even with the bottom edge of the mattress. B. Keep enough material to miter the lower mattress corners. C. Apply the drawsheet on the cleaned mattress first. D. Make the top of the bed first, moving to the bottom of the bed

A

The nurse recognizes that the older adult's progressive loss of total bone mass and tendency to take smaller steps with feet kept closer together will most likely: A. Increase the patient's risk for falls and injuries B. Result in less stress on the patient's joints. C. Decrease the amount of work required for patient movement. D. Allow for mobility in spite of the aging effects on the patient's joints

A

The nursing assessment of an 80-year-old patient who demonstrates some confusion but no anxiety reveals that the patient is a fall risk because she continues to get out of bed without help despite frequent reminders. The initial nursing intervention to prevent falls for this patient is to: A. Place a bed alarm device on the bed. B. Place the patient in a belt restraint. C. Provide one-on-one observation of the patient. D. Apply wrist restraints

A

To prevent hypoglycemia and enhance efficacy, it is appropriate to give rapid-acting insulin how many minutes before the next meal? A. 5 to 15 minutes B. 30 to 40 minutes C. 60 to 90 minutes D. The timing of insulin around meals is not necessary

A

What action might the nurse take when drawing up medication from an ampule? A. Hold the ampule upside down while inserting the filter needle B. Inject air into the ampule before withdrawing the medication C. Hold the ampule horizontally while inserting the filter needle D. Expel air bubbles from the syringe while the filter needle is still inside the ampule

A

What is the best way for the nurse to ensure that a patient receives the correct dose of a medication? A. Compare the prescriber's order with the MAR before dispensing the medication B. Ask the patient if he would like a larger dose of pain medication C. Assess the patient's ability to swallow oral medications without difficulty. D. Check the name of the mediation three times against the MAR.

A

What is the best way to minimize discomfort caused by the instillation of ear medication? A. Warm the ear drops to room temperature before instillation B. Wear treatment gloves during the application process C. Ask the patient to sit while introducing the medication D. Use a cotton-tipped applicator to remove any visible cerumen

A

What is the initial step in preparing a fecal occult blood test? A. Determine the patient's ability to help obtain a sample. B. Gather both a Hemoccult slide and developing solution. C. Provide the patient with a specimen hat or bedpan. D. Perform hand hygiene and apply treatment gloves

A

What will the nurse do first when preparing to apply PPE before caring for a patient in isolation? A. Perform hand hygiene B. Put on the gown C. Put on clean gloves D. Apply eyewear

A

What would the nurse do to remove air trapped in a syringe before withdrawing the syringe from the vial? A. Position the tip of the needle in the vial's airspace, and tap the barrel of the syringe. B. Position the tip of the needle below the fluid line, and tap the vial C. Position the vial on a flat surface, and tap the syringe D. Position the syringe above the vial, and tap the vial

A

When changing a patient's gown, the nurse will place the bed in which position? A. A comfortable working height for the nurse B. A height that allows the patient full range of motion C. Locked and low D. All side rails raised

A

When is a patient at a higher risk for a medication administration error? A. During a care transition point, such as transfer to another unit. B. While on a hospital unit for an extended length of time C. On the third postoperative day D. When taking an active role in self-administration of insulin

A

When measuring a patient's respiratory rate, the nurse will count the number of completed respiratory cycles per minute. What is the definition of a respiratory cycle? A. The number of inspirations and expirations per minute B. The number of expirations per minute C. The number of sighs per minutes D. The number of inspirations per minute.

A

When preparing an injection of mixed insulin that includes 12 units of NPH and 5 units of regular insulin, how does the nurse initially confirm the proper dosage in the syringe? A. By noting when 5 units of clear insulin is visible in the syringe B. By noting when 12 units of cloudy insulin is visible in the syringe C. By having another registered nurse verify the presence of 17 units of insulin D. By verifying that the prescription confirms the MAR

A

When preparing an injection that contains both short- and immediate-acting insulins, what is the first step the nurse would take to ensure the effectiveness of the injection? A. Insert air into the intermediate-acting insulin B. Warm the vials to room temperature C. Shake the vials to disperse the medication within the suspension D. Withdraw the prescribed amount of short-acting insulin after the intermediate-acting insulin.

A

When preparing to administer heparin or insulin subcutaneously, which site is preferred? A. Abdomen B. Scapula C. Deltoid muscle D. Back of the upper arm

A

When preparing to clean a patient's dentures using the sink, the nurse first protects the dentures by doing what? A. Padding the sink basin with a washcloth B. Performing hand hygiene C. Filling the sink with cold water D. Filling the sink with hot water

A

When preparing to provide mouth care to a patient who is in a coma, the nurse first ensures patient safety by doing what? A. Assessing the patient's gag reflex B. Inspecting the patient's oral cavity C. Placing the bed in a flat position D. Connecting the suction equipment

A

When will a patient's blood glucose levels be most affected by a short-acting insulin injection, such as Humulin-R? A. In 2 to 3 hours B. For the next 12 hours C. During unplanned exercise D. When the patient eats carbohydrates

A

Which action by the nurse is most important in protecting the safety of patients and staff when using an automated medication dispensing system? A. Refusing to share his or her individual security log-in code for the dispensing system B. Having another registered nurse check his or her mathematical calculations C. Reviewing a current drug book for dosing information D. Using two different mathematical formulas to cross-check a dosage calculation

A

Which action would best assess the effect of exercise on a patient's radial pulse measurement? A. Measuring the patient's radial pulse before and after exercise B. Assessing the patient's radial pulse 30 minutes after exercise C. Comparing the patient's radial and apical pulses after exercise D. Comparing the patient's pre-exercise radial and post-exercise apical pulses

A

Which action would take priority if a patient's apical pulse has an irregular rhythm? A. Reassess the pulse for 1 full minute B. Assess the patient's peripheral pulses C. Wait 5 minutes, and then reassess the apical pulse D. Review documentation regarding an irregular rhythm

A

Which instruction might the nurse give to NAP that is applicable only to tympanic temperature assessment? A. Leave the probe in place until the reading is complete B. Put on a new disposable probe cover for each patient C. Gently tug the pinna backward, up, and out before inserting the probe. D. Check for an impacted cerumen in the ear

A

Which instruction would help ensure the maximum therapeutic response when a patient self-administers ear medication? A. Remain in the lateral position (unaffected side) for a few minutes after instillation B. Bring refrigerated ear medication to room temperature before instillation C. Place a cotton ball firmly into the ear canal for 30 minutes after D. Apply a warm, damp washcloth to the external ear to remove any crusted discharge

A

Which instruction would the nurse give when asking nursing assistive personnel (NAP) to give a complete bed bath to a patient? A. Do not massage any reddened areas on the patient's skin. B. Be sure to wash the patient's face with soap. C. Disconnect the intravenous tubing when changing the gown. D. Wear gloves if necessary.

A

Which nursing action is most therapeutic in response to a cognitively impaired patient who demands to know when his daughter is coming to visit? A. Marking the date of the visit on the patient's wall calendar B. Evaluating the patient's understanding of the concept of time and date C. Telling the patient when his daughter will be visiting and ensuring that he verbalizes his understanding D. Calling the daughter to suggest that she visit sooner than she had planned

A

Which of the following interventions directly related to patient safety must the nurse consider when providing perineal care to an elderly male patient with a catheter? A. Wear clean gloves during care. B. Assess the patient's ability to provide self-care C. Encourage the patient to report any pain originating from the catheter. D. Monitor the amount of urine in the drainage bag to prevent overflow

A

Which of the following is a risk factor for decreased oxygen saturation level in a patient? A. Check wall injury B. Restlessness C. Hypotension D. Prescribed bronchodilators

A

Which patient is at the greatest risk for hospital-acquired infection? A. A middle-aged female patient receiving chemotherapy for lung cancer B. An older adult male patient who experienced a MI 3 days ago C. A young man recovering from bilateral femur fractures and a mild concussion sustained in a car accident D. A young woman with abdominal pain who is scheduled for exploratory surgery in the morning.

A

Which site is most commonly used for intramuscular injections? A. Ventrogluteal B. Abdominal C. Deltoid D. Dorsogluteal

A

Which statement best illustrates the nurse's understanding of the role of NAP in the use of a dry powder inhaler (DPI)? A. "Be sure to let me know if the patient starts coughing again." B. "Did you shake the inhaler well before giving it to the patient?" C. "Do you think the patient is capable of using the inhaler independently?" D. "Please tell the patient that the inhaler is to be used only when she is having trouble breathing."

A

Which statement best illustrates the nurse's understanding of the role of nursing assistive personnel (NAP) in the use of a metered-dose inhaler? A. "Be sure to let me know if she starts coughing again." B. "Show the patient how to clean the spacer chamber after she's finished with the inhaler." C. "Offer the patient her inhaler if it looks like she's short of breath." D. "Please tell her the inhaler is to be used no more than three times per day."

A

Which statement indicates proper interpretation of the results of a positive fecal occult blood test? A. "If the sample turns blue, it is positive for bleeding." B. "The sample turned blue after about 45 seconds." C. "The results were positive both times the sample was tested." D. "Because it was positive, the patient must be asked when he or she last ate red meat."

A

While reviewing a new medication order, the nurse notes that the frequency of administration has been omitted. What is the nurse's best response? A. Immediately contact the prescriber to complete the order B. Refer to a current drug book for the most commonly prescribed dosage C. Call the pharmacy to determine the frequency D. Ask a RN who is familiar with the prescriber to identify the usual frequency ordered

A

You are a new graduate nurse completing your orientation on a very busy intensive care unit. You cannot read a health care provider's order for one of your patient's medications. You have heard from more experienced nurses that this health care provider does not like to be called, and you know that another of the health care provider's patients is very unstable. What is the most appropriate next step for you to take? A. Call the health care provider to clarify the order B. Talk with your preceptor to help you interpret the order C. Refer to a medication manual before giving the medication D. Use your best judgment and critical thinking and administer the dose you think the health care provider ordered

A

You have assigned a new NAP to take routine vital signs. You notice that the NAP's last three patients have had unusually low blood pressure that you have had to confirm. What is the most likely reason the NAP is obtaining falsely low blood pressure readings? A. The blood pressure cuff is too wide for arm circumference B. The bladder was deflated too slowly C. The patient's arm was not supported while the measurement was taken D. The blood pressure cuff was not wrapped evenly around the arm

A

Fill in the blanks by typing your responses in the text box below: Current evidence shows that many nurses frequently transfer to different positions and leave the profession because of __(A)___. Implementing evidence-based interventions and programs reduces the number of ___(B)___, which improves the health of the nurse and reduces ____(C)___ to the health care agency.

A) Work-related injuries B) Work-related injuries C) indirect costs

A nurse is teaching a community group about ways to minimize the risk of developing osteoporosis. Which of the following statements reflect understanding of what was taught? (Select all that apply.) A. "I usually go swimming with my family at the YMCA 3 times a week." B. "I need to ask my doctor if I should have a bone mineral density check this year." C. "If I don't drink milk at dinner, I'll eat broccoli or cabbage to get the calcium that I need in my diet." D. "I'll check the label of my multivitamin. If it has calcium, I can save money by not taking another pill." E. "My lactose intolerance should not be a concern when considering my calcium intake."

A, B, C

A patient who has been isolated for C.DIFF asks you to explain what he should know about this organism. What is the most appropriate information to include in patient teaching? (select all that apply) A. The organism is usually transmitted through the fecal-oral route B. Hands should always be cleaned with soap and water versus alcohol-based sanitizer C. Everyone coming into the room must be wearing a gown and gloves D. While the patient is in contact precautions, he cannot leave the room E.. C.diff dies quickly once outside the body

A, B, C

The body alignment of the patient in the tripod position includes the following: (Select all that apply.) A. An erect head and neck B. Straight vertebrae C. Extended hips and knees D. Axillae resting on the crutch pads E. Bent knees and hips

A, B, C

The infection control nurse has asked the staff to work on reducing the number of iatrogenic infections on the unit. Which of the following actions on your part would contribute to reducing health care-acquired infections? (select all that apply) A. Teaching correct handwashing to assigned patients B. Using correct procedures in starting and caring for an intravenous infusion C. Providing perineal care to a patient with an indwelling urinary catheter D. Isolating a patient who has just been diagnosed as having TB E. Decreasing a patient's environment stimuli to decrease nausea

A, B, C

A nurse is educating parents to look for clues in teenagers for possible substance abuse. Which environmental and psychosocial clues should the nurse include? (Select all that apply.) A. Blood spots on clothing B. Long-sleeved shirts in warm weather C. Changes in relationships D. Wearing dark glasses indoors E. Increased computer use

A, B, C, D

Before transferring a patient from the bed to a stretcher, which assessment data do the nurse need to gather? (Select all that apply.) A. Patient's weight B. Patient's level of cooperation C. Patient's ability to assist D. Presence of medical equipment E. Nutritional intake

A, B, C, D

A couple who is caring for their aging parents are concerned about factors that put them at risk for falls. Which factors are most likely to contribute to an increase in falls in the elderly? (Select all that apply.) A. Inadequate lighting B. Throw rugs C. Multiple medications D. Doorway thresholds E. Cords covered by carpets F. Staircases with handrails

A, B, C, D, E

You are conducting an education class at a local senior center on safe-driving tips for seniors. Which of the following should you include? (Select all that apply.) A. Drive shorter distances B. Drive only during daylight hours C. Use the side and rearview mirrors carefully D. Keep a window rolled down while driving if has trouble hearing E. Look behind toward the blind spot F. Stop driving at age 75

A, B, C, D, E

A nurse is evaluating a patient who is in soft wrist restraints. Which of the following activities does the nurse perform? (Select all that apply.) A. Check the patient's peripheral pulse in the restrained extremity B. Evaluate the patient's need for toileting C. Offer the patient fluids if appropriate D. Release both limbs at the same time to perform range of motion (ROM) E. Inspect the skin under each restraint

A, B, C, E

What is your role as a nurse during a fire? (Select all that apply.) A. Help to evacuate patients B. Shut off medical gases C. Use a fire extinguisher D. Single carry patients out E. Direct ambulatory patients

A, B, C, E

The nurse enters the room of an 82 year old patient for whom she has not care previously. The nurse notices that the patient wears a hearing aid. The patient looks up as the nurse approaches the bedside. Which of the following approaches are likely to be effective with an older adult? (select all that apply) A. Listen attentively to the patient's story B. Use gestures that reinforce your questions or comments C. Stand back away from the bedside D. Maintain direct eye contact E. Ask questions quickly to reduce the patient's fatigue

A, B, D

The effects of immobility on the cardiac system include which of the following? (Select all that apply.) A. Thrombus formation B. Increased cardiac workload C. Weak peripheral pulses D. Irregular heartbeat E. Orthostatic hypotension

A, B, E

Which of the following patients are at most risk for tachypnea? (Select all that apply.) A. Patient just admitted with four rib fractures B. Woman who is 9 months' pregnant C. Adult who has consumed alcoholic beverages D. Adolescent waking from sleep E. Three-pack-per-day smoker with pneumonia

A, B, E

Which type of PPE are staff required to wear when caring for a pediatric patient who is placed into airborne precautions for confirmed chickenpox/herpes zoster? (select all that apply) A. Disposable gown B. N 95 respirator mask C. Face shield or goggles D. Surgical mask E. Gloves

A, B, E

A nurse reviews data gathered regarding a patient's ability to cope with loss. The nurse compares the defining characteristics for Ineffective Coping with those for Readiness for Enhanced Coping and selects Ineffective Coping as the correct diagnosis. This is an example of the nurse avoiding an error in: (select all that apply) A. Data collection B. Data clustering C. Data interpretation D. Making a diagnostic statement E. Goal setting

A, C

A patient has an indwelling urinary catheter. Why does an indwelling urinary catheter present a risk for UTI? (select all that apply) A. It allows migration of organisms into the bladder B. The insertion procedure is not done under sterile procedure conditions C. It obstructs the normal flushing action of urine flow D. It keeps an incontinent patient's skin dry E. The outer surface of the catheter is not considered sterile

A, C

Review the following problem-focused nursing diagnoses and identify the diagnoses that are stated correctly. (select all that apply) A. Impaired skin integrity related to physical immobility B. Fatigue related to heart disease C. Nausea related to gastric distention D. Need for improved Oral Mucosa Integrity related to inflamed mucosa E. Risk for infection related to surgery

A, C

Which strategies should a nurse use to facilitate a safe transition of care during a patient's transfer from the hospital to a skilled nursing facility? (Select all that apply.) A. Collaboration between staff members from sending and receiving departments B. Requiring that the patient visit the facility before a transfer is arranged C. Using a standardized transfer policy and transfer tool D. Arranging all patient transfers during the same time each day E. Relying on family members to share information with the new facility

A, C

A nurse knows that the people most at risk for accidental hypothermia are: (Select all that apply.) A. People who are homeless. B. People with respiratory conditions. C.People with cardiovascular conditions D. The very old E. People with kidney disorders

A, C, D

A patient is receiving 5000 units of heparin subcutaneously every 12 hours while on prolonged bed rest to prevent thrombophlebitis. Because bleeding is a potential side effect of this medication, the nurse should continually assess the patient for the following signs of bleeding: (Select all that apply.) A. Bruising B. Pale yellow urine C. Bleeding gums D. Coffee ground-like vomitus E. Light brown stool

A, C, D

The American Dental Association suggests that patients who are at risk for poor hygiene use the following interventions for oral care: (Select all that apply.) A. Use antimicrobial toothpaste. B. Brush teeth 4 times a day. C. Use 0.12% chlorhexidine gluconate (CHG) oral rinses. D. Use a soft toothbrush for oral care. E. Avoid cleaning the gums and tongue

A, C, D

When working with an older adult who is hearing-impaired, the use of which techniques would improve communication? (Select all that apply.) A. Check for needed adaptive equipment. B. Exaggerate lip movements to help the patient lip read. C. Give the patient time to respond to questions. D. Keep communication short and to the point. E. Communicate only through written information.

A, C, D

After receiving an intramuscular (IM) injection in the deltoid, a patient states, "My arm really hurts. It's burning and tingling where I got my injection. What should the nurse do next? (Select all that apply.) A. Assess the injection site B. Administer an oral medication for pain C. Notify the patient's health care provider of assessment findings D. Document assessment findings and related interventions in the patient's medical record E. This is a normal finding so nothing needs to be done F. Apply ice to the site for relief of burning pain

A, C, D,

A nurse is getting ready to assess a patient in a neighborhood community clinic. He was newly diagnosed with diabetes just a month ago. He has other health problems and a history of not being able to manage his health. Which of the following questions reflects the nurse's cultural competence in making an accurate diagnosis? (select all that apply) A. How is your diabetic diet affecting you and your family? B. You seem to not want to follow health guidelines. Can you explain why? C. What worries you the most about having diabetes? D. What do you expect from us when you do not take your insulin as instructed? E. What do you believe will help you control your blood sugar?

A, C, E

Motivational interviewing (MI) is a technique that applies understanding a patient's values and goals in helping the patient make behavior changes. What are other benefits of using MI techniques? (Select all that apply.) A. Gaining an understanding of patient's motivations B. Focusing on opportunities to avoid poor health choices C. Recognizing patient's strengths and supporting their efforts D. Providing assessment data that can be shared with families to promote change E. Identifying differences in patient's health goals and current behaviors

A, C, E

The use of standard formal nursing diagnostic statements serves several purposes in nursing practice, including which of the following? (select all that apply) A. Defines a patient's problem, giving members of the health care team a common language for understanding the patient's needs B. Allows physicians and allied health staff to communicate with nurses how they provide care among themselves C. Helps nurses focus on the scope of nursing practice D. Creates practice guidelines for collaborative health care activities E. Builds and expands nursing knowledge

A, C, E

A nurse makes the following statement during a change-of-shift report to another nurse. "I assess Mr. Diaz, my 61 year old patient from Chile. He fell at home and hurt his back 3 days ago. He has some difficulty turning in bed, and he says that he has pain that radiates down his leg. He rates his pain at a 6, and he moves slowing as he transfers to a chair?" What can the nurse who is beginning a shift do to validate the previous nurse's assessment findings when she conducts rounds on the patient? (select all that apply) A. The nurse asks the patient to rate his pain on a scale of 0 to 10 B. The nurse asks the patient what caused his fall C. The nurse asks the patient if he has had pain in his back in the past D. The nurse assess the patient's lower-limb strength E. The nurse asks the patient what pain medication is most effective in managing his pain

A, D

An 88-year-old patient comes to the medical clinic regularly. During a recent visit the nurse noticed that the patient had lost 10 lbs in 6 weeks without being on a special diet. The patient tells the nurse that he has had trouble chewing his food. Which of the following factors are normal aging changes that can affect an older adult's oral health? (Select all that apply.) A. Dentures do not always fit properly. B. Most older adults have an increase in saliva secretions. C. With aging the periodontal membrane becomes tighter and painful. D. Many older adults are edentulous, and remaining teeth are often decayed. E. The teeth of elderly patients are more sensitive to hot and cold

A, D

A nurse gathers the following assessment data. Which of the following cues together form(s) a pattern suggesting a problem? (select all that apply) A. The skin around the wound is tender to touch B. Fluid intake for 8 hours is 800mL C. Patient has heart rate of 78 beats/min and regular D. Patient has drainage from surgical wound E. Body temperature is 38.3C (101F) G. Patient states, "I'm worried that i won't be able to return to work when I planned."

A, D, E

In which of the following examples are nurses making diagnostic errors? (select all that apply) A. The nurse who observes a patient wincing and holding his left side and gathers no additional assessment data B. The nurse who measures joint range of motion after the patient reports pain in the left elbow C. The nurse who considers conflicting cues in deciding which diagnostic label to choose D. The nurse who identifies a diagnosis on the basis of a patient reporting difficulty sleeping E. The nurse who makes a diagnoses of Ineffective Airway Clearance related to pneumonia

A, D, E

A healthy adult patient tells the nurse that he obtained his blood pressure in "one of those quick machines in the mall" and was alarmed that it was 152/72 when his normal value ranges from 114/72 to 118/78. The nurse obtains a blood pressure of 116/76. What would account for the blood pressure of 152/92? (Select all that apply.) A. Cuff too small B. Arm positioned above heart level C. Slow inflation of the cuff by the machine D. Patient did not remove his long-sleeved shirt E. Insufficient time between measurements

A, E

11. A nurse is teaching a community group about ways to minimize the risk of developing osteoporosis. Which of the following statements reflect understanding of what was taught? (Select all that apply.) a. "I usually go swimming with my family at the YMCA 3 times a week." b. "I need to ask my doctor if I should have a bone mineral density check this year." c. "If I don't drink milk at dinner, I'll eat broccoli or cabbage to get the calcium that I need in my diet." d. "I'll check the label of my multivitamin. If it has calcium, I can save money by not taking another pill." e. "My lactose intolerance should not be a concern when considering my calcium intake."

A,B,C

26. The body alignment of the patient in the tripod position includes the following: (Select all that apply.) a. An erect head and neck b. Straight vertebrae c. Extended hips and knees d. Axillae resting on the crutch pads e. Bent knees and hips

A,B,C

Which of the following examples are steps of nursing assessment? (Select all that apply) A. Collection of information from patient's family members B. Recognition that further observations are needed to clarify information C. Comparison of data with another source to determine data accuracy D. Determining which medications to administer based on a patient's assessment data

A,B,C

28. Before transferring a patient from the bed to a stretcher, which assessment data do the nurse need to gather? (Select all that apply.) a. Patient's weight b. Patient's level of cooperation c. Patient's ability to assist d. Presence of medical equipment e. Nutritional intake

A,B,C,D

71. Which skills must a patient with a new colostomy be taught before discharge from the hospital? (Select all that apply.) a. How to change the pouch b. How to empty the pouch c. How to open and close the pouch d. How to irrigate the colostomy e. How to determine if the ostomy is healing appropriately

A,B,C,E

42. The nurse is caring for a patient who exhibits labored breathing, using accessory muscles, and is coughing up pink frothy sputum. The patient has bilateral lung bases and diminished breath sounds. What are the priority nursing assessments for the nurse to perform prior to notifying the patient's health care provider? (Select all that apply.) a. SpO2 levels b. Amount, color and consistency of sputum production c. Fluid status d. Change in respiratory rate and pattern e. Pain in lower leg

A,B,D

14. The effects of immobility on the cardiac system include which of the following? (Select all that apply.) a. Thrombus formation b. Increased cardiac workload c. Weak peripheral pulses d. Irregular heartbeat e. Orthostatic hypotension

A,B,E

73. Which of the following symptoms are warning signs of possible colorectal cancer according to the American Cancer Society guidelines? (Select all that apply.) a. Change in bowel habits b. Blood in the stool c. A larger-than-normal bowel movement d. Fecal impaction e. Muscle aches f. Incomplete emptying of the colon g. Food particles in the stool h. Unexplained abdominal or back pain

A,B,F,H

12. A patient is receiving 5000 units of heparin subcutaneously every 12 hours while on prolonged bed rest to prevent thrombophlebitis. Because bleeding is a potential side effect of this medication, the nurse should continually assess the patient for the following signs of bleeding: (Select all that apply.) a. Bruising b. Pale yellow urine c. Bleeding gums d. Coffee ground-like vomitus e. Light brown stool

A,C,D

55. The nurse is educating the patient and his family about the parenteral nutrition. Which aspect related to this form of nutrition would be appropriate to include? (Select all that apply.) a. The purpose of the fat emulsion in parenteral nutrition is to prevent a deficiency in essential fatty acids. b. We can give you parenteral nutrition through your peripheral intravenous line to prevent further infection. c. The fat emulsion will help control hyperglycemia during periods of stress. d. The parenteral nutrition will help your wounds heal. e. Since we just started the parenteral nutrition, we will only infuse it at 50% of your daily needs for the next 6 hours.

A,C,D

57. The nurse is teaching a program on healthy nutrition at the senior community center. Which points should be included in the program for older adults? (Select all that apply.) a. Avoid grapefruit and grapefruit juice, which impair drug absorption. b. Increase the amount of carbohydrates for energy. c. Take a multivitamin that includes vitamin D for bone health. d. Cheese and eggs are good sources of protein. e. Limit fluids to decrease the risk of edema

A,C,D

When a nurse conducts an assessment, data about a patient often comes from which of the following sources? (select all that apply) A. An observation of how a patient turns and moves in bed B. The unit policy and procedure manual C. The care recommended of a physical therapist D. The results of a diagnostic x-ray film E. Your experience in caring for other patients with similar problems

A,C,D

41. A patient was admitted following a motor vehicle accident with multiple fractured ribs. Respiratory assessment includes signs/symptoms of secondary pneumothorax. Which are the most common assessment findings associated with a pneumothorax? (Select all that apply). a. Sharp pleuritic pain that worsens on inspiration b. Crackles over lung bases of affected lung c. Tracheal deviation toward the affected lung d. Worsening dyspnea e. Absent lung sounds to auscultation on affected side

A,D,E

70. Which are key points that the nurse should include in patient education for a person with complaints of chronic constipation? (Select all that apply.) a. Increase fiber and fluids in the diet b. Use a low-volume enema daily c. Avoid gluten in the diet d. Take laxatives twice a day e. Exercise for 30 minutes every day f. Schedule time to use the toilet at the same time every day g. Take probiotics 5 times a week

A,E,F

What is one of the major functions of the large intestine? Break down food Absorption of water Make bowel sounds Absorb nutrients

Absorption of water

Mr. Doe has a nasogastric (NGT) tube and is receiving enteral nutrition and reports he is constipated, what intervention would most likely help? Add water flushes Increasing the rate of the tube feeds Adding protein to the formula Decreasing the rate of the tube feed

Add water flushes

What important intervention must be done to alleviate diarrhea in a patient receiving enteral feedings? Administer antibiotic per m.d. order Fill the bag all the way to the top Do nothing it is normal to have diarrhea when receiving enteral feedings Avoid contamination of formula by changing the feeding bag every 24 hours

Avoid contamination of formula by changing the feeding bag every 24 hours

. Before discharge, the nurse shows a patient how to use a dry powder inhaler. What should the nurse now assess? A. Patient's understanding of the purpose of the medication B. Patient's ability to handle, manipulate, and activate the DPI. C. Adequacy of the patient's planning daily medication schedule D. Patient's awareness of the signs of an allergic reaction to the medication

B

. Which of the following is not taken into consideration when determining the appropriate amount of a topical medication to be applied to the skin? A. Size of the skin site B. Other medications the patient is taking C. Manufacturer's instructions for application of the product D. Health care provider's order

B

101. What would the nurse do first when preparing to begin oxygen therapy for a patient? a. Educate the NAP about the oxygen orders. b. Review the medical prescription for delivery method and flow rate. c. Place a "No Smoking" sign outside of the hospital room. d. Ensure that suction equipment is present in the room.

B

107. What would the nurse do first to ease breathing for a patient with mild dyspnea? a. Administer oxygen at 2 L/min by nasal cannula. b. Help the patient into an upright sitting position. c. Monitor the patient's pulse oximetry level. d. Determine if the patient has a history of respiratory pathology.

B

11. When repositioning a patient, what can the nurse do to prevent the patient's hips from rolling outward? a. Apply therapeutic boots to the feet. b. Place sandbags along the legs c. Place a small pillow at the lumbar region of the back. d. Place a pillow under the calves.

B

110. What would the nurse do routinely to monitor oxygenation in a patient receiving BiPAP? a. Assess the patient's level of consciousness every 4 hours. b. Monitor the patient's pulse oximetry readings. c. Verify the pressure settings for both inspiratory and expiratory pressure. d. Evaluate daily arterial blood gases (ABGs)

B

117. The nurse is discussing the guidelines for proper hand hygiene with NAP. Which statement made by NAP requires follow-up by the nurse? A. "I always perform hand hygiene after I use the computer workstation in the patient's room." B. "To prevent dry skin, I avoid using soap and water." C. "It takes at least 15 seconds of rubbing to wash the hands properly." D. "I do hand hygiene before and after lifting and moving my patients."

B

13. When positioning a hemiplegic patient in the supported Fowler's position, what is the primary reason a trochanter roll is placed alongside the patient's legs? a. To reduce the risk of a fall while the side rails are down b. To reduce the risk of contracture c. To control pain d. To cushion the legs

B

17. Which of the following most motivates a patient to participate in an exercise program? a. Providing a patient with a pamphlet on exercise b. Providing information to the patient when he or she is ready to change behavior c. Explaining the importance of exercise at the time of diagnosis of a chronic disease d. Providing the patient with a booklet with examples of exercises e. Providing the patient with a prescribed exercise program

B

19. A patient lying supine in bed is being transferred to a wheelchair using a transfer belt. Which action would the nurse perform just before moving the patient to the side of the bed? a. Help the patient put on skid-resistant footwear b. Raise the head of the bed 30 degrees c. Place the transfer belt over the patient's clothing d. Position the chair so that the patient ill move toward his or her strong side

B

2. The nurse is preparing to initiate ambulation with a patient who is recovering from a stroke. What information will help the nurse determine how far to walk? a. Ask the patient how far she would like to go b. Review the health care provider's order c. Review the medical record to see how far the patient has walked during the past several therapeutic ambulations d. Review records of other patients who are at a similar point in their stroke rehabilitation

B

21. Which is the correct gait when a patient is ascending stairs on crutches? a. A modified two-point gait (The affected leg is advanced between the crutches to the stairs.) b. A modified three-point gait (The unaffected leg is advanced between the crutches to the stairs.) c. A swing-through gait d. A modified four-point gait. (Both legs advance between the crutches to the stairs.)

B

22. A patient with a right knee replacement is prescribed no weight bearing on the right leg. You reinforce crutch walking knowing that which of the following crutch gaits is most appropriate for this patient? a. Two-point gait b. Three-point gait c. Four-point gait d. Swing-through gait

B

25. The nurse is preparing to use a slide board to transfer a patient from the bed to a stretcher. How many additional people will the nurse need to help with this transfer? a. 4 b. 2 c. 1 d. none

B

26. When preparing to delegate the application of the SCD to NAP, the nurse must do what first? a. Ask the NAP to demonstrate the proper application of the SCD. b. Assess the patient's lower extremities for signs and symptoms of impaired circulation c. Assess the patient's need for a SCD d. Explain that the device is not to be removed

B

27. The nurse has applied the SCD to a postoperative patient. The most appropriate way for the nurse to confirm proper fit is to do what? a. Ask the patient if the device is caused any pain b. Ensure that two fingers will fit between the patient's leg and the device c. Follow the manufacturer's instructions for the application of the device d. Ask another nurse to check the patients for proper application of the device

B

29. Why might a sequential compression device (SCD) be applied ot the legs of an immobile patient? a. To stimulate circulation in the deep arterial vascular system b. To help prevent deep vein thrombosis (DVT) c. To aide peripheral circulation to reduce the risk of skin breakdown d. To assist in passive range-of-motion exercise of the patient's lower extremities

B

3. The nurse is preparing to delegate the ambulation of a patient with the use of a gait belt to nursing assistive personnel (NAP). Which statement made by NAP requires the nurse to follow up? a. "I will be sure to put nonskid slippers on the patient before getting him up to ambulate" b. "I will use the under-axillae technique to help him up to a standing position" c. "rocking the heavier patient into a standing position seems to work really well for me" d. "I will grasp the gait belt in the middle of the patient's back

B

31. A patient is admitted to the emergency department with suspected carbon monoxide poisoning. Even though her color is ruddy not cyanotic, the nurse understands the patient is at a risk for decreased oxygen-carrying capacity of blood because carbon Monoxide does which of the following: a. Stimulates hyperventilation causing respiratory alkalosis b. Forms a strong bond with hemoglobin thus preventing oxygen binding in the lungs c. Stimulates hypoventilation causing respiratory acidosis d. Causes alveoli to overinflate leading to atelectasis

B

33. After determining the proper size stocking and assessing the patient's circulatory status, a nurse delegates the application of elastic stockings to nursing assistive personnel (NAP). The nurse discovers that the NAP has been using moisturizer on the patient's legs before applying the stockings. What is the best action by the nruse? a. Explain that moisturizer may cause excessive skin softening, which can lead to skin breakdown. b. Instruct NAP to use a small amount of cornstarch or powder c. Ask the patient if he or she is allergic to the moisturizer d. Inspect the patient's skin for color variations

B

34. The nurse is caring for a patient who has decreased mobility. Which intervention is a simple and cost-effective method for reducing the risks of pulmonary complication? a. Antibiotics b. Frequent change of position c. Oxygen humidification d. Chest physiotherapy

B

39. Which patient is most at risk of developing permanently impaired mobility? a. A 72-year-old woman hospitalized for anemia associated with diabetic nephropathy (kidney disease) b. A 55-year-old woman with mental illness who had become malnourished c. An 11-year-old boy who sustained a fractured pelvis during a fall from his tree house d. A 79-year-old man recovering from surgery to release a contracture of the connective tissue in her hand

B

40. The nurse notes that a patient's left elbow is resistant to extension and flexion while performing range of motion exercises. What is the appropriate nursing action? a. Move the joint through the full range of motion exercises b. Perform range of motion to the left elbow until resistance is met c. Omit all the range of motion exercises until the health care provider is notified d. Inform the health care provider that the patient is uncooperative with exercising.

B

41. The nurse is delegating to nursing assistive personnel (NAP) the administration of an enema for an older adult patient who is recovering from a stroke. The enema order reads, "Enemas until clear." Which statement made by NAP requires the nurse to follow-up? a. "I'll need help to turn her onto her side" b. "It may take three or four enemas to achieve a clear return" c. "I'll test the water temperature on the inside of my own wrist." d. "The enema will wear her out, so I'll wait until after she ambulates."

B

43. Which action would the nurse take to in ensure the safety of an older adult patient who has received an enema? a. Assess for the presence of external hemorrhoids. b. Provide assistance to the bathroom for expulsion of fluid and stool. c. Document the patient's physical response to the enema. d. Instruct the patient to attempt to retain the fluid for 2 to 5 minutes.

B

43. Which of the following skills can be delegated to nursing assistive personnel (NAP)? (Select all that apply.) a. Nasotracheal suctioning b. Oropharyngeal suctioning of a stable patient c. Suctioning a new artificial airway d. Permanent tracheostomy tube suctioning e. Care of an endotracheal tube (ETT)

B

46. After assisting with a bedpan, the nurse notes that the patient's stool is streaked with bright-red blood. What would the nurse do first? a. Notify the patient's health care provider b. Ask if the patient has a history of hemorrhoids. c. Check the medical record to see if the patient has a history of blood in the stool d. Document the observation in the medical record, indicating a need for follow-up

B

47. A dependent, confused patient is being given a bedpan. How can the nurse best ensure the patient's safety? a. Respond promptly to the call light. b. Raise the side rails on the bed before leaving the room c. Slide one hand under the patient's sacrum to help the patient lift off the bedpan. d. Check in on the patient every 5 minutes until the bedpan can be removed.

B

48. The nurse has delegated to nursing assistive personnel (NAP) the skill of assisting with a bedpan for patient who has had discomfort when walking to the bathroom. Which statement made by the NAP requires the nurse's follow-up? a. "Do you still need a stool sample for the lab?" b. "If I can get someone to help, I'll walk her to the bathroom." c. "The patient reports that moving is uncomfortable for he. Has she had pain medication recently?" d. "The patient told me that she's had problems with hemorrhoids in the past."

B

5. The nurse is helping a patient with hemiparesis take a few steps. A gait belt has been applied. The patient is using a cane. Where should the nurse stand in relation to the patient? a. On the patient's strong side b. On the patient's weak side c. Behind the patient d. In front of the patient

B

52. The nurse sees the nursing assistive personnel (NAP) perform the following intervention for a patient receiving continuous enteral feedings. Which action would require immediate attention? a. Fastening tube to the gown with new tape b. Placing patient supine while giving a bath c. Hanging a new container of enteral feeding d. Ambulating patient with enteral feedings still infusing

B

53. A patient is receiving total parenteral nutrition (TPN). What is the primary intervention the nurse should follow to prevent a central line infection? a. Institute isolation precautions b. Clean the central line port through which the TPN is infusing with alcohol c. Change the TPN tubing every 24 hours d. Monitor glucose levels to watch and assess for glucose intolerance

B

53. The nurse is delegating to nursing assistive personnel (NAP) the task of assisting with a urinal. The nurse specifies to NAP that the urinal is to be used in bed, not in a standing position, for the patient? a. Patient admitted for hypertension and diabetes b. Patient with complete left-sided paralysis caused by a stroke c. Patient receiving diagnostic tests for esophageal strictures d. Patient being treated for dehydration from heat exposure

B

55. The nurse is assisting a patient with the placement of a urinal. The patient tells the nurse, "I'll call you when I'm done." What is the nurse's best response? a. "All right, my name is Robin, and I'll be right across the hall. Just call me when you're finished" b. "Fine. Recap the urinal, hang it on your side rail, and use your call light toilet me know you're finished." c. "I'll check on you as soon as I get a chance." d. "I'll be back in 15 minutes. That should be enough time for you to finish up."

B

56. Which statement best illustrates the nurse's understanding of the role of nursing assistive personnel (NAP) is carrying out a gastric occult blood test for a patient with a low hemoglobin and hematocrit? a. "Have you used the new Gastroccult testing system?" b. "The next time the patient vomits, please test it for occult blood." c. "Is the patient capable of assisting with the specimen collection?" d. "Remember to tell me the results of the test immediately."

B

58. Which statemtn best illustrates correct interpretation of a postitive gastric occult blood test? a. "We don't need to retest the patient right now, because the sample turned green after about 60 seconds." b. "If the test sample turns blue, it is positive for blood." c. "The monitor area needs to turn blue within 30 seconds." d. "Because it was positive, I notified the patient's physician."

B

59. The nurse should do which of the following when placing a bedpan under an immobilized patient? a. Lift the patient's hips off the bed and slide the bedpan under the patient b. After positioning the patient on the bedpan, elevate the head of the bed to a 45-degree angle c. Adjust the head of the bed so it is lower than the feet and use gentle but firm pressure to push the bedpan under the patient d. Have the patient stand beside the bed and then have him or her sit on the bedpan on the edge of the bed

B

6. To prevent complications of immobility, what would be the most effective activity on the first postoperative day for a patient who has had abdominal surgery? a. Turn, cough, and deep breathe every 30 minutes while awake b. Ambulate patient to chair in the hall c. Passive range of motion 4 times a day d. Immobility is not a concern the first postoperative day

B

62. A patient is told the home care nurse will be measuring and recording intake and output (I&O) at home. What will the home care nurse do first? a. Supply a urine hat b. Explain to the patient why I&O has been ordered. c. Assess the patient's ability to self-monitor and record I&O. d. Provide the patient's family with instructions.

B

67. An elderly patient comes to the hospital with a complaint of severe weakness and diarrhea for several days. Of the following problems, which is the most important to assess initially? a. Malnutrition b. Dehydration c. Skin breakdown d. Incontinence

B

68. A female nurse is preparing to administer a rectal suppository to a male patient. The patient says, "this isso embarrassing. Is this really necessary?" What is the most appropriate response? a. "I can see if the doctor with order an oral medication" b. "How about if I show you how to insert the suppository yourself?" c. "I will make sure that you are well covered. I promise." d. "This will make you feel so much better."

B

7. When preparing to move a patient in bed, what will the nurse do first? a. Assemble adequate help to move the patient b. Assess the patient's ability to help with moving c. Determine the patient's weight d. Decide on the most effective means of moving the patient

B

71. What instruction might the nurse give to nursing assistive personnel (NAP) regarding postoperative exercises? a. "Find out if the patient has any language barriers." b. "Let me know when the patient actually begins exercising." c. "Please review a copy of the preoperative literature with the patient." d. "Assess the method of learning the patient would prefer."

B

73. Why might a nurse teach a patient scheduled for surgery how to do postoperative exercises? a. To maximize a sense of well-being b. To minimize postoperative complications c. To identify cultural factors that reflect the patient's perception of pain d. To evaluate the patient's ability to participate in postoperative activities

B

74. Before teaching a patient postsurgical exercises, the nurse premedicates the patient for pain. What benefit does this have specific to the patient's learning? a. Reduced pain b. Improved focus c. Decreased relaxation d. Decreased irritability

B

75. Which instruction might a nurse give a patient in order to protect a surgical incision when turning in bed? a. Hold your breath when turning. b. Use a pillow to splint the incision. c. Take pain medication 30 minutes before turning. d. Keep both legs straight when turning.

B

80. What would the nurse do if he or she encountered resistance when inserting a nasogastric tube? a. Ask the patient to cough. b. Withdraw the tube to the nasopharynx. c. Encourage the patient to swallow. d. Instruct the patient to hyperextend the neck

B

82. What would the nurse use to irrigate a patient's nasogastric tube after providing medications? a. Coffee b. Purified water c. Tea d. Apple juice

B

86. What would the nurse monitor frequently to ensure that the prescribed amount of oxygen is being delivered to a patient? a. Arterial blood gas (ABG) levels b. Oxygen flow meter setting c. Respiratory rate d. Temperature

B

94. Which instruction might the nurse give to nursing assistive personnel (NAP) regarding the care of a patient with a newly established colostomy? a. "Be sure to pat-dry the skin surrounding the stoma before applying the new pouch." b. "Alert me immediately if you see any blood in the fecal matter in the pouch." c. "Using the stoma guide, cut the pouch opening about one-eighth of an inch bigger than the stoma." d. "Remember to change your gloves after cleaning the stoma and the surrounding skin."

B

A nurse is talking with a young-adult patient about the purpose of a new medication. The nurse says, "I want to be clear. Can you tell me in your words the purpose of this medicine?" This exchange is an example of which element of the transactional communication process? A. Message B. Obtaining feedback C. Channel D. Referent

B

A nurse prepares to contact a patient's physician about a change in the patient's condition. Using SBAR (Situation, Background, Assessment, and Recommendation) communication, which of the following is the correct order? 1."She is a 53-year-old female who was admitted 2 days ago with pneumonia and was started on Levaquin at 5 pm yesterday. She complains of a poor appetite." 2. "The patient reported feeling very nauseated after her dose of Levaquin an hour ago." 3. "Would you like to make a change in antibiotics, or could we give her a nutritional supplement before her medication?" 4. "The patient started complaining of nausea yesterday evening and has vomited several times during the night." A. 1, 3, 4, 2 B. 4, 1, 2, 3 C. 2, 1, 3, 4 D. 4, 2, 1, 3

B

A nursing student is administering ampicillin PO. The expiration date on the medication wrapper was yesterday. What is the appropriate action for the nursing student to take next? A. Ask the nursing professor for advice B. Return the medication to pharmacy and get another tablet C. Call the health care provider after discussing this situation with the charge nurse D. Administer the medication since medications are good for 30 days after their expiration date

B

A nursing student is working with a faculty member to identify a nursing diagnosis for an assigned patient. The student has assessed that the patient is undergoing radiation treatment and has had liquid stool and the skin is clean and intact; therefore she selects the nursing diagnosis Impaired Skin Integrity. The faculty member explains that the student has made a diagnostic error for which of the following reasons? A. Incorrect clustering B. Wrong diagnostic label C. Condition is a collaborative problem D. Premature closure of clusters

B

A patient is diagnosed with MRSA pneumonia. Which type of isolation precaution is most appropriate for this patient? A. Reverse isolation B. Droplet precautions C. Standard precautions D. Contact precautions

B

A patient with a right knee replacement is prescribed no weight bearing on the right leg. You reinforce crutch walking knowing that which of the following crutch gaits is most appropriate for this patient? A. Two-point gait B. Three-point gait C. Four-point gait D. Swing-through gait

B

After instructing a patient in the self administration of antibiotic eardrops, what must come first in the nurse's assessment? A. The patient's understanding of the medication's purpose B. The patient's hand grasp, strength, coordination, and ability to manipulate the applicator C. The patient's comprehension of the dosage instructions provided with the medication D. The patient's ability to recognize the signs of an allergic reaction to the medication

B

After instructing a patient in the self-administration of antibiotic eye drops, what is the nurse's highest priority assessment? A. The patient's understanding of the medication's purpose B. The patient's hand grasp, strength, coordination, and ability to manipulate the applicator C. The patient's comprehension of the dosage instructions provided with the medication D. The patient's ability to recognize the signs of an allergic reaction to the medication

B

After seeing a patient, the health care provider starts to give a nursing student a verbal order for a new medication. The nursing student first needs to: A. Follow ISMP guidelines for safe medication abbreviations B. Explain to the health care provider that the order needs to be given to a registered nurse. C. Write down the order on the patient's order sheet and read it back to the health care provider. D. Ensure that the six rights of medication administration are followed when giving the medication.

B

As a nurse is preparing to provide perineal care to a female patient with limited mobility, the patient says, "I can do that myself." Which action would be the priority? A. Provide all the necessary supplies and linen for this task B. Assess the patient's ability to perform proper perineal care C. Ensure that the patient has privacy while performing perineal care D. Document any complaints of irritation or pain in the perineal area

B

As the nurse is at the bedside preparing to administer a new medication, the patient mentions that he is allergic to the drug. What will the nurse do first? A. Notify the physician B. Withhold the medication C. Check to see if the patient is wearing a red allergy ID band D. Review the MAR for allergies

B

During the admissions process, the nurse initially assesses the patient's radial pulse primarily for what purpose? A. Assessment of peripheral blood perfusion B. Establishment of a baseline as part of the patient's vital signs C. Assessment of the patient's cardiovascular disease risk D. Determination of oxygen saturation

B

For which situation would the procedure of glucose testing be interrupted? A. The reagent strip code matches the code on the vial B. An unused lancet is not available C. The glucose meter beeps D. A drop of blood forms on the patient's skin after it is punctured

B

How does soap work? A. Creates a sheetlike microscopic barrier between the skin and microorganisms in the environment B. Emulsifies fat and oil so that dirt and microorganisms can be mechanically removed C. Kills all microorganisms exposed to the lather D. Removes visible soiling but is ineffective in isolating, removing, or killing microorganisms

B

In providing foot care, the nurse would soak the feet and hands of which patient? A. A 30-year-old man with type 1 diabetes B. An 86-year-old woman with generalized weakness C. A 56-year-old patient with vascular insufficiency who was bathed the day before D. A 56-year-old patient with vascular insufficiency who was not bathed the day before

B

The nurse asks a patient, "Describe for me a typical night's sleep. What do you do to fall asleep? Do you have difficulty falling or staying asleep?" This series of questions would likely occur during which phase of a patient-centered interview? A. Orientation B. Working phase C. Data validation D. Termination

B

The nurse delegates needed hygiene care for an elderly stroke patient. Which intervention would be appropriate for the nurse assistant to accomplish during the bath? A. Checking distal pulses B. Providing range-of-motion (ROM) exercises to extremities C. Determining type of treatment for stage 1 pressure ulcer D. Changing the dressing over an intravenous site

B

The nurse has delegated to nursing assistive personnel (NAP) the task of performing fecal occult blood tests on the stool of a patient with a history of positive results. Which instruction is most relevant to performing this test in this particular patient? A. "Notify me only if the test is positive." B. "Save the stool sample so that I can retest it if it is positive." C. "Remind the patient that we test one section of the bowel movement." D. "Use Gastroccult developer with Hemoccult developer."

B

The nurse is assisting a patient with a tub bath. After the patient has been safely positioned in the tube, he tells the nurse, "I'll call you when I'm done." What is the nurse's best response? A. "All right. Just holler when you're ready, and I'll come and help you get out of the tub." B. "Well, I'll check back with you in about 5 minutes to see if you need anything." C. "That's not safe. I'll wait right outside the door for you to finish." D. "I'll be back in 15 minutes. That should be enough time for you to finish up."

B

The nurse is changing the bed linen of a patient on bed rest. When the nurse is ready to make the other side of the bed, what will the nurse do before having the patient turn onto the side that has already been made? A. Lower the head of the bed. B. Raise the side rails C. Apply the topsheet D. Discard the soiled linen in the linen bag

B

The nurse is delegating a female patient's perineal care to NAP. Which instruction would the nurse give to ensure the NAP's safety while performing this care? A. Wear sterile gloves B. Wear clean gloves C. Wear an isolation gown D. Use hot water

B

The nurse is discussing the guidelines for proper use of PPE by NAP. Which statement made by the NAP requires follow-up by the nurse? A. "When in doubt, I wear gloves." B. "I really dislike wearing the mask, so it's the first thing I take off." C. "I always do hand hygiene when entering and leaving a patient's room." D. "I wear a mask whenever I am caring for a patient who's coughing"

B

The nurse is instructing a patient who is to receive both a bronchodilator and a steroid medication delivered by means of a metered-dose inhaler (MDI). Which instruction is necessary for the safe administration of both agents? A. "Make sure to use the steroid medication before the bronchodilator." B. "Make sure to use the bronchodilator before the steroid medication." C. "Rinse your mouth with warm water before using the MDI to administer either medication." D. "Make sure you wait at least 30 seconds between administering the bronchodilator and administering the steroid medication."

B

The nurse is planning to measure a patient's blood pressure. What does the systolic measurement represent? A. Minimal pressure on the arterial walls B. The pressure exerted against the arterial wall C. The change in pressure from a lying to a sitting position D. The last sound heard when measuring the blood pressure

B

The nurse is preparing to administer several oral medications when the patient says he would like to take his pills with orange juice. What is the nurse's best response? A. Determine whether the patient's prescribed diet includes orange juice B. Establish whether the medications may be taken with orange juice C. Ask the dietary aide to order extra orange juice for the unit D. Administer the pills with orange juice

B

The nurse is preparing to assess a patient's blood pressure. What would cause the blood pressure reading to be inaccurately high? A. Blood pressure cuff is too wide B. Blood pressure cuff is too loose around the arm C. Taking the blood pressure in an arm into which intravenous fluids are infusing D. Arm is positioned above the level of the heart

B

The nurse is preparing to give a patient a bath using a disposable bath-in-a-bag product. What should the nurse do first? A. Remove the patient's gown. B. Warm the product in the microwave. C. Obtain a washbasin. D. Gather towels and washcloths.

B

The nurse is preparing to mix short- and intermediate-acting insulins to administer to a patient. Which action best preserves the insulin's effectiveness? A. Determining the patient's blood glucose level B. Refraining from injecting the intermediate-acting insulin into the short-acting vial C. Applying clean gloves when administering the medication D. Having another registered nurse verify the dose of both types of insulins

B

The nurse wants to offer some diversional activity to a patient with dementia. The patient's family has told the nurse that he is a bit of a loner who enjoyed a 40-year career as an aircraft mechanic. The patient seems frustrated and bored. What is the best activity for the nurse to offer him? A. Weekly put therapy with a golden retriever. B. A jigsaw puzzle of an appropriate level of difficulty C. A crossword puzzle book of an appropriate level of difficulty D. Frequent card games with other patients

B

The nursing diagnoses Impaired Parenting related to mother's developmental delay is an example of a(n): A. Risk nursing diagnoses B. Problem-focused nursing diagnosis C. Health promotion nursing diagnosis D. Wellness nursing diagnosis

B

The patient is to receive both Lantus (insulin glargine) and regular insulin. To ensure the proper action of the insulins, what would the nurse do when preparing these two types of insulin for administration? A. Mix the insulins in one syringe for a single injection B. Prepare the insulins in two syringes for separate injections C. Roll each vial between the palms to disperse the medication within the suspension D. Have another registered nurse verify the dose of the insulins

B

The patient refuses a scheduled dose of an antibiotic, saying that the medication makes him feel nauseated. What is the nurse's best response? A. Inform the patient why the medication is necessary B. Notify the prescriber of the patient's reason for refusing the medication C. Offer to administer the medication with the patient's favorite snack food D. Note the patient's refusal in the MAR

B

To prevent complications of immobility, what would be the most effective activity on the first postoperative day for a patient who has had abdominal surgery? A. Turn, cough, and deep breathe every 30 minutes while awake B. Ambulate patient to chair in the hall C.Passive range of motion 4 times a day D. Immobility is not a concern the first postoperative day

B

To which patient might the nurse apply a physical restraint? A. An 83-year-old patient with dementia and a history of wandering whose fall risk assessment indicates a high risk of falling. B. A 42-year-old critical care patient with a traumatic brain injury who has repeatedly tried to pull out her shunt. C. A 74-year-old patient confined to bed who is at risk of pressure ulcers D. A 60-year-old patient with dementia who seems increasingly confused shortly after having had restraints applied for 1 hour that morning.

B

What can the nurse do to help ensure an accurate result when collecting a midstream urine sample for a patient who is menstruating? A. Notify the health care provider B. Make a note on the lab slip that the patient is menstruating C. Postpone the specimen collection until menses has ceased. D. Do nothing other than follow normal procedure, since menstruation will not affect the results.

B

What is the best way for the nurse to minimize the risk of contaminating the patient's eye during the instillation of eye drops? A. Encourage the patient to self-apply the medication B. Wear gloves during the entire application process C. Introduce the medication onto the inner canthus of the eye D. Perform effective hand hygiene before and after the instillation

B

What is the most appropriate way for the nurse to split an unscored tablet? A. Use a pill-splitting device to split the unscored pill in half B. Ask the pharmacy if it is appropriate to split the pill and if so, ask them to split and repackage it with the adjusted dose given on the label C. Use scissors to cut the pill in half D. Administer a whole pill every other day instead of every day

B

What is the most important action the nurse can take to ensure that a midstream urine specimen does not become contaminated? A. Wear sterile gloves to open the sterile specimen kit B. Ensure that the patient's perineum has been cleansed before the specimen is obtained. C. Determine if the patient has any known allergies D. Have the patient rate his or her current pain level

B

What is the nurse's best response after noticing that the route of administration has been omitted from a medication order? A. Ask which route the patient prefers. B. Immediately notify the prescriber to request that the order be completed C. Refer to a current drug book to determine the most commonly prescribed route. D. Contact the pharmacy to determine the most appropriate route for this patient.

B

What is the nurse's first response when a patient requests another dose of narcotic pain medication before it is time for the next dose? A. Consult with the physical therapy department to arrange for a visit with the patient B. Work with the patient to find alternative nonpharmacologic means of pain management C. Contact the patient's provider to request an order for additional pain medication D. Give the patient a detailed explanation of the need to limit the amount of narcotic medication she takes

B

What is the nurse's first step in preparing to administer a prescribed medication using an automated medication dispensing system? A. Establish the patient's ID using two identifiers. B. Review the medication administration record (MAR). C. Provide patient education. D. Review applicable nursing considerations

B

What is the nurse's priority action if a patient's radial pulse has an irregular rhythm? A. Reassess the pulse for 1 full minute B. Assess the patient for a pulse deficit C. Wait 5 minutes, and then reassess the pulse. D. Review documentation regarding an irregular rhythm

B

What is the primary purpose of initially assessing an apical pulse? A. Assessment of the patient's cardiac function B. Establishment of a baseline as part of the patient's vital signs C. Assessment of the patient's risk for cardiovascular disease D. Determination of oxygen saturation

B

What must the nurse avoid when brushing the tongue of an unconscious patient? A. Dislodging bacteria B. Stimulating the gag reflex C. Moistening the oral mucosa D. Using suction

B

What should the nurse teach NAP about selecting the appropriate site for measuring a patient's oxygen saturation level? A. "Do not use the fingers if her nails are polished" B. "I've checked her capillary refill, and it's acceptable in both hands and feet." C. "Please review the patient's previously documented pulse oximetry readings for the site used." D. "Ask the patient to keep her fingers motionless while you are monitoring her oxygen saturation."

B

What statement made by a 24-year-old patient's mother indicates that she understands how to administer her son's eardrops? A. "To straighten his ear canal, I need to pull the outside part of his ear down and back." B. "I need to straighten his ear canal before administering the medication by pulling his ear upward and outward." C. "I need to put my son in a chair and make sure that he's sitting up with his head tilted back before I give him the eardrops." D. "After I'm done giving him his eardrops, I need to make sure that my son remains sitting straight up for at least 10 minutes."

B

What would the nurse instruct NAP to report when caring for a patient in a wrist restraint? A. "Tell me if the patient's pulse changes." B. "Tell me if the skin under the restraint becomes abraded or raw." C. "Let me know if you think she's ready for them to come off." D. "Let me know if the patient needs anything for pain."

B

What would the nurse instruct NAP to report when performing denture care for a patient? A. The amount of time it takes to clean the patient's dentures B. The appearance of any cracks in the dentures C. Any dietary preferences of the patient that could affect the teeth D. Whether the patient uses mouthwash.

B

What would the nurse instruct the NAP to do before making an unoccupied bed if the mattress is soiled? A. Wash the mattress with hot water. B. Wipe off moisture with antiseptic solution, and dry thoroughly C. Flip the mattress D. Apply a waterproof pad over the soiled area

B

When a NAP enters the room of a patient in a belt restraint, he finds the patient's gown bunched around the patient's chest and the patient asking for help. What would the NAP do? A. Check the patient's blood pressure and pulse before smoothing the gown B. Untie the restraint and smooth the patient's gown C. Put on the call light for help D. Ask the patient what specific help she would like

B

When administering an intradermal injection, which outcome would require the nurse to withdraw the needle and begin again? A. Aspiration of blood prior to injecting the medication B. Inability to feel resistance when injecting the medication C. Formation of a 6-mm bleb at the injection site D. Appearance of a lesion resembling a mosquito bite at the injection site

B

When placing an intraocular disk, the nurse recognizes that it is in the correct position by assessing what? A. Visibility of the disk over the cornea B. Lack of visibility of the disk as it is placed under the lower eyelid C. Lack of visibility as it is placed under the upper eyelid D. Visibility of a small portion of the disk extending slightly above the lower eyelid

B

When preparing to administer a new medication, what would the nurse do first to ensure the patient's safety? A. Perform hand hygiene B. Compare the written order with the MAR C. Inform the patient about the medication D. Review appropriate nursing considerations

B

When preparing to assist a patient with a gown change, the nurse will promote infection control by doing what after performing hand hygiene? A. Placing the gown in the linen hamper B. Applying clean gloves C. Determining whether the gown being changed is wet or soiled D. Measuring the patient's temperature to determine whether it is elevated

B

Where should the nurse measure the blood pressure of a patient recovering from a left-sided mastectomy? A. Use the left arm to take the blood pressure B. Use the right arm to take the blood pressure C. Do not take the blood pressure D. Use a lower extremity to take the blood pressure

B

Which action by the nurse ensures patient safety when administering an intramuscular injection? A. Putting on clean gloves before administration B. Rotating injection sites C. Aspirating for blood return when administering a vaccine D. Injecting the medication quickly

B

Which action can the nurse take to keep a patient from consciously controlling his or her breathing during an assessment? A. Take a patient's temperature while counting the respiratory rate B. Assess respiration after measuring the pulse C. Assess respiration after taking the blood pressure D. Assess respiration before measuring the blood pressure

B

Which action would the nurse encourage an older adult with foot problems to take at home? A. Apply oval pads to treat corns B. Wear socks made of natural fibers C. Carefully shave off calluses with a razor blade D. If a bandage is needed, apply gauze squares with adhesive tape

B

Which action would the nurse take to diminish tissue irritation when administering a subcutaneous injection to a patient of average size? A. Massage the site after administration B. Make sure the volume of the medication is less than 2 mL C. Administer the injection at a 45- to 90-degree angle D. Wear clean gloves while administering the injection

B

Which discharge instruction would help to ensure that the patient achieves maximum therapeutic delivery of the medication when using a metered-dose inhaler (MDI)? A. Make sure to report any adverse effects after using your inhaler B. Prime the inhaler if it is new or has not been used for several days C. Hold your breath for 60 seconds after the medication is delivered D. Use the inhaler while sitting up in a chair at 90-degree angle

B

Which instruction would be given to a patient to ensure safety when self-applying an antibiotic ointment? A. It is not necessary to allow refrigerated eye medication to warm to room temperature before administration B. Do not apply pressure directly to the eyeball when removing excess medication C. When cleaning the eye before administration, gently wash from the outer to the inner canthus D. Apply a warm, damp washcloth to the eye for several minutes to remove any crusted discharge

B

Which is the correct gait when a patient is ascending stairs on crutches? A. A modified two-point gait (The affected leg is advanced between the crutches to the stairs.) B. A modified three-point gait (The unaffected leg is advanced between the crutches to the stairs.) C. A swing-through gait D. A modified four-point gait. (Both legs advance between the crutches to the stairs)

B

Which of the following discharge instructions would be most important in ensuring the safety of a patient who will need to apply a dermal patch daily at home? A. Apply sufficient pressure to the edges of the patch to ensure adequate adherence B. Avoid using a heating pad on or near the application site C. Pat the application site dry before applying the patch D. Reapply the patch to the same site each time to enhance absorption

B

Which of the following is contraindicated with taking a rectal temperature measurement? A. Patient requires assistance to move to a side-lying position B. Patient has painful and swollen hemorrhoids C. Patient is incontinent of urine D. The last temperature recorded was 0.2F above baseline

B

Which of the following most motivates a patient to participate in an exercise program? A. Providing a patient with a pamphlet on exercise B. Providing information to the patient when he or she is ready to change behavior C. Explaining the importance of exercise at the time of diagnosis of a chronic disease D. Providing the patient with a booklet with examples of exercises E. Providing the patient with a prescribed exercise program

B

Which of the following nursing actions will reduce the risk of "wrong route" when administering a medication? A. Only splitting pills or tablets that have been prescored by the manufacturer B. Using an oral dosing syringe when administering oral liquid medication C. Transcribing a fraction dose of less than one with a leading zero (e.g., 0.5 mg) D. Crushing an oral medication that is difficult to swallow

B

Which of the following statements made by an older adult reflects the best understanding of the need to exercise regardless of age? A. "You are never too old to begin an exercise program." B. "My grandaughter and I walk together around the high school track 3 times a week." C. "I purchased a subscription to a runner's magazine for my grandson for Christmas." D. "When I was a child, I exercised more than I see kids doing today."

B

Which of the following statements made by an older adult reflects the best understanding of the need to exercise regardless of age? a. "You are never too old to begin an exercise program." b. "My granddaughter and I walk together around the high school track 3 times a week." c. "I purchased a subscription to a runner's magazine for my grandson for Christmas." d. "When I was a child, I exercised more than I see kids doing today."

B

Which patient should not have his or her feet soaked during a complete bed bath? A. A patient with arthritis B. A patient with diabetes mellitus C. A patient who has just complained of shoulder pain D. A patient who is nauseated

B

Which statement best illustrates the nurse's understanding of appropriate sites for the application of an estrogen patch? A. "I'll check to see if the patient has pendulous breasts." B. "I need to assess the skin on the patient's thighs." C. "I need to encourage her to wear elastic waistbands." D. "I'll tell her to wear blouses and shirts with loose sleeves."

B

Which statement best illustrates the nurse's understanding of the role of NAP in documenting medicaiton administration? A. "Make a note that the patient just received her PM dose of pain medication." B. "Let me know if the patient says her nausea is getting worse." C. "Can you check the MAR and see when this patient had her pain medication last?" D. "Ask the patient if I need to get another order from the provider."

B

Which statement might the nurse make to nursing assistive personnel (NAP) in order to help ensure reliable results of culture and sensitivity testing of a midstream urine specimen? A. "I'll need a biohazard bag to put the specimen into." B. "Please get the specimen to the lab within 20 minutes." C. "After you replace the cap, please wipe any drops of urine from the outside of the container." D. "We are out of specimen collection kits."

B

Which statement or question best illustrates the nurse's understanding of the role of NAP in administering oral medications? A. "Does the patient need her pain medication?" B. "Please make sure the patient has plenty of fresh water to take with her pills." C. "How much did the pain medication improve her pain?" D. "Stay with the patient until he swallows all the pills."

B

Which statement or question best illustrates the nurse's understanding of the role of NAP in medication administration? A. "Does the patient need her pain medication?" B. "Let me know if she complains of any nausea." C. "What is the quality of her pain now?" D. "Tell her she doesn't have an order for the drug she's asking for."

B

Which statement or question best illustrates the nurse's understanding of the role of nursing assistive personnel (NAP) in using automated medication dispensing systems? A. "Does the patient need her pain medication?" B. "Let me know if she complains of any nausea." C. "Ask her to describe her pain and show you where it is." D. "Remember to log off of the system when you are finished."

B

Which type of interview question does the nurse first use when assessing the reason for patient seeking health care? A. Probing B. Open-ended C. Problem-oriented D. Confirmation

B

Why must the nurse check the flow rate after changing the gown of a patient who is receiving intravenous fluids infused by gravity? A. To make sure the patient's position change has not sealed off the drop chamber B. To check whether manipulation of the intravenous container and tubing has disrupted the flow rate C. To see if an increase in the patient's heart rate has accelerated the flow rate D. To make the standard periodic flow rate adjustment

B

The nurse evaluates that the NAP has applied a patient's sequential compression device (SCD) appropriately when which of the following is observed? (Select all that apply.) A. Initial patient measurement is made around the calves B. Inflation pressure averages 40 mm Hg C. Patient's leg placed in SCD sleeve with back of knee aligned with popliteal opening on the sleeve. D. Stockings are removed every 2 hours during application. E. Yellow light indicates SCD device is functioning.

B, C

The nurse uses silence as a therapeutic communication technique. What is the purpose of the nurse's silence? (Select all that apply.) A. Prevent the nurse from saying the wrong thing B. Prompt the patient to talk when he or she is ready C. Allow the patient time to think and gain insight D. Allow time for the patient to drift off to sleep E. Determine if the patient would prefer to talk with another staff member

B, C

What is the proper position to use for an unresponsive patient during oral care to prevent aspiration? (Select all that apply.) A. Prone position B. Sims position C. Semi-fowler's position with head to side D. Trendelenburg position E. Supine position

B, C

Which of the following actions by the nurse comply with core principles of surgical asepsis? (select all that apply) A. Set up sterile field before patient and other staff come to the operating suite B. Keep the sterile field in view at all times C. Consider the outer 2.5cm (1 inch) of the sterile field as contaminated D. Only health care personnel within the sterile field must wear PPE E. The sterile gown must be put on before the surgical scrub is performed

B, C

An older adult has limited mobility as a result of a total knee replacement. During assessment you note that the patient has difficulty breathing while lying flat. Which of the following assessment data support a possible pulmonary problem related to impaired mobility? (Select all that apply.) A. B/P = 128/84 B. Respirations 26/min on room air C. HR 114 D. Crackles over lower lobes heard on auscultation E. Pain reported as 3 on scale of 0 to 10 after medication

B, C, D

Nurses must communicate effectively with the health care team for which of the following reasons? (Select all that apply.) A. Improve the nurse's status with the health team members B. Reduce the risk of errors to the patient C. Provide optimum level of patient care D. Improve patient outcomes E. Prevent issues that need to be reported to outside agencies

B, C, D

A nurse is assigned to a new patient admitted to the nursing unit following admission through the ER. The nurse collects a nursing history and interviews the patient. Place the following steps for making a nursing diagnosis in the correct order, beginning with the first step. A. Considers context of patient's health problem and selects a related factor B. Reviews assessment data, noting objective and subjective clinical information C. Clusters clinical cues that form a pattern D. Chooses diagnostic label

B, C, D, A

You are working in a health clinic on a college campus. You need to administer medroxyprogesterone acetate intramuscularly (IM) to a female patient for birth control. You look up this medication in a reference manual and determine that it is viscous and injections can be painful. On the basis of this information, you plan which of the following when administering this medication? (Select all that apply.) A. Inject the medication over 3 minutes to reduce pain associated with the injection B. Administer the medication in the ventral gluteal site C. Use the z-track method when administering the medication D. Use the deltoid site for medication administration E. Ask the patient questions about her major and which classes she is taking during the injection to provide distraction

B, C, E

What does it mean when a patient is diagnosed with a multidrug resistant organism in his or her surgical wound? (select all that apply) A. There is more than one organism in the wound that is causing the infection B. The antibiotics the patient has received are not strong enough to kill the organism C. The patient will need more than one type of antibiotic to kill the organism D. The organism has developed a resistance to one or more broad-spectrum antibiotics, indicating that the organism will be hard to treat effectively E. There are no longer any antibiotics options available to treat the patient's infection

B, D

Which of the following nursing diagnoses is stated correctly? (select all that apply) A. Fluid Volume Excess related to heart failure B. Sleep Deprivation related to sustained noisy environment C. Impaired Bed Mobility related to postcardiac catheterization D. Ineffective Protection related to inadequate nutrition E. Diarrhea related to frequent, small, watery, stools

B, D

A nurse is conducting a patient-centered interview. Place the statements from the interview in the correct order, beginning with the first statement a nurse would ask. A. "You say you've lost weight. Tell me how much weight you've lost in the last month." B. "My name is Todd. I'll be the nurse taking care of you today. I'm going to ask you a series of questions to gather your health history." C. "I have no further questions. Thank you for your patience." D. "Tell me what brought you to the hospital." E. "So, to summarize, you've lost about 6lbs in the last month, and your appetite has been poor-correct?"

B, D, A, E, C

A patient is admitted for dehydration caused by pneumonia and shortness of breath. He has a history of heart disease and cardiac dysrhythmias. The nursing assistant reports his admitting vital signs to the nurse. Which measurements should the nurse reassess? (Select all that apply.) A. Right arm BP: 118/72 B. Radial pulse rate: 72 and irregular C. Temporal temperature: 37.4° C (99.3F) D. Respiratory rate: 28 E. Oxygen saturation: 99%

B, D, E

The nurse is caring for a patient who is having a seizure. Which of the following measures will protect the patient and the nurse from injury? (Select all that apply.) A. If patient is standing, attempt to get him or her back in bed. B. With patient on floor, clear surrounding area of furniture or equipment. C. If possible, keep patient lying supine. D. Do not restrain patient; hold limbs loosely if they are flailing. E. Never force apart a patient's clenched teeth.

B, D, E

The student nurse is teaching a family member the importance of foot care for his or her mother, who has diabetes. Which safety precautions are important for the family member to know to prevent infection? (Select all that apply.) A. Cut nails frequently. B. Assess skin for redness, abrasions, and open areas daily. C. Soak feet in water at least 10 minutes before nail care. D. Apply lotion to feet daily. E. Clean between toes after bathing.

B, D, E

When should a nurse wear a mask? (select all that apply) A. The patient's dental hygiene is poor B. The nurse is assisting with an aerosolizing respiratory procedure such as suctioning C. The patient has acquired immunodeficiency syndrome (AIDS) and a congested cough D. The patient is in droplet precautions E. The nurse is assisting a health care provider in the insertion of a central line catheter

B, D, E

The nursing assistive personnel (NAP) reports to you that the blood pressure (BP) of the patient in Question 11 is 140/76 on the left arm and 128/72 on the right arm. What actions do you take on the basis of this information? (Select all that apply.) A. Notify the health care provider immediately B. Repeat the measurements on both arms using a stethoscope C. Ask the patient if she has taken her blood pressure medications recently D. Obtain blood pressure measurements on lower extremities E. Verify that the correct cuff size was used during the measurements F. Review the patient's record for her baseline vital signs G. Compare right and left radial pulses for strength

B, F

13. The nurse evaluates that the NAP has applied a patient's sequential compression device (SCD) appropriately when which of the following is observed? (Select all that apply.) a. Initial patient measurement is made around the calves b. Inflation pressure averages 40 mm Hg c. Patient's leg placed in SCD sleeve with back of knee aligned with popliteal opening on the sleeve. d. Stockings are removed every 2 hours during application. e. Yellow light indicates SCD device is functioning.

B,C

56. The nurse would delegate which of the following to nursing assistive personnel (NAP)? (Select all that apply.) a. Repositioning and retaping a patient's nasogastric tube b. Performing glucose monitoring every 6 hours on a patient c. Documenting PO intake on a patient who is on a calorie count for 72 hours d. Administering enteral feeding bolus after tube placement has been verified e. Hanging a new bag of enteral feeding

B,C

72. Which of the following may cause Clostridium difficile infection? (Select all that apply.) a. Chronic laxative use b. Contact with C. difficile bacteria c. Overuse of antibiotics d. Frequent episodes of diarrhea caused by food intolerance e. Inflammation of the bowel

B,C

10. An older adult has limited mobility as a result of a total knee replacement. During assessment you note that the patient has difficulty breathing while lying flat. Which of the following assessment data support a possible pulmonary problem related to impaired mobility? (Select all that apply.) a. B/P = 128/84 b. Respirations 26/min on room air c. HR 114 d. Crackles over lower lobes heard on auscultation e. Pain reported as 3 on scale of 0 to 10 after medication

B,C,D

54. The nurse is caring for a patient with pneumonia who has severe malnutrition. The nurse recognizes that, because of the nutritional status, the patient is at increased risk for: (Select all that apply.) a. Heart disease. b. Sepsis. c. Pleural effusion. d. Cardiac arrhythmias. e. Diarrhea.

B,C,D

58. Which patients are at high risk for nutritional deficits? (Select all that apply.) a. The divorced computer programmer who eats precooked food from the local restaurant b. The middle-age female with celiac disease who does not follow her gluten-free diet c. The 45-year-old patient with type II diabetes who monitors her carbohydrate intake and exercises regularly d. The 25-year-old patient with Crohn's disease who follows a strict diet but does not take vitamins or iron supplements e. The 65-year-old patient with gallbladder disease whose electrolyte, albumin, and protein levels are normal

B,D

A colostomy or ileostomy stoma should be all the following except. Black Red Moist Pink

Black

. A nurse is preparing to withdraw medication from an open multi-dose vial. After confirming that the vial contains the appropriate medication and check the expiration date, what would the nurse do next? A. Apply clean gloves B. Vigorously shake the vial C. Wipe the rubber seal of the vial with an alcohol swab D. Introduce air equal to the amount of medication needed.

C

10. A patient who weighs 200 lbs. needs to be moved up in bed with the aid of a friction-reducing device. The nurse will prepare for this move by assembling how many caregivers a. A minimum of 2 b. None, since the device does all the lifting during the move c. At least 3 d. The nurse can carry out this move without assistance

C

102. When preparing the patient's environment for safe oxygen therapy, which intervention is a priority to minimize the patient's risk for injury? a. Place appropriate signage to alert staff and visitors to the presence of oxygen in the patient's room. b. Instruct nursing assistive personnel (NAP) to immediately correct or report safety hazards. c. Inspect all electrical equipment in the patient's room for the presence of safety-check tags. d. Ensure that the patient receives the prescribed amount of oxygen via the appropriate method.

C

103. Which statement by the patient would indicate that he or she understands the safe use of oxygen? a. "The nurse told me that my oxygen saturation must be maintained at 85% or above." b. "I know that oxygen is a medication I can adjust whenever I need to." c. "I'll alert the nurse immediately if I have any increased difficulty breathing." d. "I often experience difficulty breathing for no apparent reason, but that is expected."

C

104. When a patient is receiving oxygen at home, which instruction to the family would help them understand how to use the oxygen safely? a. Increase the oxygen level as needed for the patient's comfort. b. Store extra oxygen cylinders horizontally. c. Place a "No Smoking" sign at the entrance to the house. d. Keep oxygen 5 feet (about 1.5 meters) away from anything that could generate a spark.

C

109. What is the purpose of splinting the abdomen with a small pillow during controlled coughing? a. To minimize chest discomfort caused by the coughing b. To expand lung capacity during the inspiratory phase of the cough c. To maximize transdiaphragmatic pressure during the expiratory phase of the cough d. To focus the patient's attention on the abdominal muscles used during the cough

C

15. A nurse is instructing a patient who has decreased leg strength on the left side how to use a cane. Which action indicates proper cane use by the patient? a. The patient keeps the cane on the left side of the body. b. The patient slightly leans to one side while walking. c. The patient keeps two points of support on the floor at all times. d. After the patient places the cane forward, he or she then moves the right leg forward to the cane.

C

15. The nurse is preparing to move a patient with hemiplegia into the prone position. What action should the nurse take when rolling the patient onto her side? a. Place a small pillow under her shoulder. b. Use the affected arm as a guide during rolling c. Place a pillow on the abdomen. d. Place rolled bath blankets along the dependent leg

C

16. When preparing to safely transfer a patient from a bed to a wheelchair using a transfer belt, the nurse would do what first? a. Coordinate extra help. b. Assess the patient's vital signs c. Assess the patient's physiological capacity to transfer d. Determine whether to transfer the patient to a wheelchair or chair

C

18. A patient has been transferred to a wheelchair with a transfer belt. What is one action the nurse would take to position the patient safely in the chair? a. Remove the wheelchair leg rests. b. Ask the patient to rate his or her pain level c. Lower the foot rests, and place the patient's feet on them d. Remove the transfer belt

C

19. A nurse assistive personnel asks for help to transfer a patient who is 125 lbs (56.8 kg) from the bed to a wheelchair. The patient is unable to help. What is the nurse's best response? a. "As long as we use proper body mechanics, no one will get hurt." b. "The patient only weighs 125 lbs. You don't need my assistance." c. "Call the lift team for additional assistance." d. "The two of us can lift the patient easily."

C

22. The nurse is preparing to move a patient from the bed to a stretcher. What will the nurse do first? a. Cross the patient's arms over his or her chest. b. Lower the side rails of the bed c. Make sure the bed brakes are locked d. Fanfold the drawsheet

C

23. Which of the following indicates that additional assistance is needed to transfer the patient from the bed to the stretcher? a. The patient is 5 feet 6 inches and weighs 120 lbs. b. The patient speaks and understands English. c. The patient received an injection of morphine 30 minutes ago for pain. d. You feel comfortable handling a patient of this size and level of cooperation

C

24. After moving a patient from the bed to a stretcher, the nurse raises the head of the stretcher. What will the nurse do next? a. Lock the wheels on the stretcher. b. Cover the patient with a blanket c. Raise the side rails on the stretcher d. Unlock the wheels of the bed.

C

25. Musculoskeletal disorders are the most prevalent and debilitating occupational health hazards for nurses. To reduce the risk for these injuries, the American Nurses Association is advocating which of the following? a. Mandate that physical therapists do all patient transfers b. Require adequate staffing levels in health care organizations c. Require the use of assistive equipment and devices d. Require an adequate number of staff to be involved in all patient transfers

C

30. While preparing to apply a SCD for a postoperative patient, the nurse realizes that which assessment observation contraindicates the application of the device? a. Have a low-grade fever b. Taking a prescribed anticoagulant c. Having dermatitis on the legs d. Wearing elastic stockings

C

31. Which condition is not associated with venous stasis, part of Virchow's triad? a. Pregnancy b. Obesity c. Anxiety d. Immobility

C

34. Why might the nurse choose not to apply a pair of prescribed elastic stockings to a patient's legs? a. The patient will have a scheduled bath in a few hours. b. The patient says they are too tight. c. The patient's skin is irritated d. The patient has become fully ambulatory

C

35. A patient is admitted with severe lobar pneumonia. Which of the following assessment findings would indicate that the patient needs airway suctioning? a. Coughing up thick sputum only occasionally b. Coughing up thin, watery sputum after nebulization c. Decreased ability to clear airway through couching d. Lung sounds clear only after coughing

C

37. Which of the following are basic guidelines when assisting a patient with passive range of motion? a. Exercises should be continued until the point of fatigue and pain b. Exercises should be done frequently to lessen pain for the patient c. Each joint is exercised to the point of resistance but not pain d. Exercises should be performed without the support to each joint

C

44. Which action would the nurse take to reduce the risk of infection among patients and staff when administering an enema to an older adult patient with dementia? a. Lubricate the tip of the rectal tube. b. Pad the patient's bed thoroughly c. Perform hand hygiene before donning globes d. Help the patient onto a bedpan to expel the enema fluid and stool

C

5. A patient on prolonged bed rest is at an increased risk to develop this common complication of immobility if preventive measures are not taken: a. Myoclonus b. Pathological fractures c. Pressure ulcers d. Pruritus

C

50. A patient with a nasogastric tube, an intravenous infusion line, and an indwelling urinary catheter needs to be placed on the bedpan. Which action would the nurse take first to ensure the patient's safety? a. Close the bedside curtain b. Raise the side rail on the side opposite that on which the nurse is working c. Obtain help to place the patient on the bedpan d. Raise the bed to a comfortable working height

C

50. The patient's blood glucose level is 330 mg/dL. What is the priority nursing intervention? a. Recheck by performing another blood glucose test. b. Call the primary health care provider. c. Check the medical record to see if there is a medication order for abnormal glucose levels. d. Monitor and recheck in 2 hours.

C

51. Which action promotes infection control when assisting a patient with a urinal? a. Placing a clean urinal on the overbed table b. Using a waterproof pad to protect the linen from urine spillage c. Applying gloves before emptying and cleaning the patient's urinal d. Asking if the patient would like to clean the genitals after using the urinal

C

52. Why would the nurse assess a patient's abdomen before helping with the use of a urinal? a. To determine if the patient needs a bed pan for bowl elimination b. To assess for abdominal pain c. To assess for bladder distention d. To determine if the patient will need help using the urinal

C

59. Why might the nurse delegate to nursing assistive personnel (NAP) the skill of performing a gastric occult blood test for a patient who has vomited? a. The task is easy to demonstrate to NAP b. The likelihood of a positive result is minimal c. This skill may be delegated if performed on vomited stomach contents d. The agency transin NAP to perform only NG tube testing

C

60. What is the initial step in preparing to perform a gastric occult blood test for a patient with recurrent vomiting? a. Determine the patients ability to help obtain the specimen b. Gather a gastroccult slide and developing solution c. Review the medications the patient is currently taking d. Perform hand hygiene, and apply treatment gloves.

C

62. The nurse is teaching the patient to obtain a specimen for fecal occult blood testing using fecal immunochemical (FIT) testing at home. How does the nurse instruct the patient to collect the specimen? a. Three fecal smears from one bowel movement b. One fecal smear from an early-morning bowel movement c. One fecal smear from three separate bowel movements d. Three fecal smears when blood can be seen in the bowel movement

C

63. A patient is admitted to your unit for dehydration. Which of the following assessments would the nurse identify as a possible sign of fluid imbalance a. Heart rate at 80 beats per minute b. Capillary refill of less than 2 seconds c. Reduced turgor of the skin d. B/P of 118/78 mmHg

C

63. When a patient has fecal incontinence as a result of cognitive impairment, it may be helpful to teach caregivers to do which of the following interventions? a. Cleanse the skin with antibacterial soap and apply talcum powder to the buttocks b. Use diapers and heavy padding on the bed c. Initiate bowel or habit training program to promote continence d. Help the patient to toilet once every hour

C

64. Which statement reflects the nurse's understanding of the importance of accurate urinary output measurement for a patient with acute renal failure? a. "If the output begins to decrease, I will notify the physician immediately." b. "Increasing his fluid intake both orally and intravenously should boost his urine output." c. "I will use a collectin system with an hourly measurement device added." d. "I will explain to the patient and family why I&O is being measured and recorded."

C

65. The nurse is caring for a patient with an ileostomy. Which intervention is most important? a. Cleansing the stoma with hot water b. Inserting a deodorant tablet in the stoma bag c. Selecting or cutting a pouch with an appropriate-size stoma opening d. Wearing sterile gloves while caring for the stoma

C

65. What output will the nurse direct nursing assistive personnel (NAP) to measure for a hospitalized patient for whom I&O measurement is prescribed? a. Nasogastric tube drainage b. Chest tube drainage c. Urine collection drainage d. Ileostomy bag drainage

C

66. The nurse is taking a health history of a newly admitted patient with a diagnosis of possible fecal impaction. Which of the following is the priority question to ask the patient or caregiver? a. Have you eaten more high-fiber foods lately? b. Are your bowel movements soft and formed? c. Have you experienced frequent, small liquid stools recently? d. Have you taken antibiotics recently?

C

66. Which statement made by a nurse best illustrates an understanding of the role of nursing assistive personnel (NAP) in administering a rectal suppository? a. "Find out whether the patient is capable of inserting the suppository" b. "Please tell the patient to report if any rectal bleeding occurs." c. "Be sure to let me know if the patient has a bowel movement." d. "Remember to lubricate the suppository."

C

72. The nurse is concerned that a patient will not be able to turn independently in bed after having surgery. What must the nurse do to help this patient? a. Reinstruct the patient in proper turning techniques. b. Document that the patient refuses to turn independently. c. Communicate that the staff must turn the patient after surgery. d. Restrict turning unless absolutely necessary.

C

76. What would the nurse do if he or she were not able to insert a nasogastric tube in either of a patient's nares? a. Ask another nurse to attempt the insertion. b. Document the attempts in the patient's medical record. c. Notify the physician that the attempts were unsuccessful. d. Allow the patient to rest for 30 minutes before resuming the process.

C

8. When preparing to move a patient in bed with the help of an assistant, which posture will give caregivers use to ensure their own safety a. Stand with the knees locked b. Stand with the feet together c. Flex the hips and knees d. Shift the body weight from the back leg to the front leg

C

81. What would the nurse do before providing the patient with a scheduled intermittent feeding through a nasogastric (NG) tube? a. Listen to bowel sounds. b. Listen to lung sounds. c. Check NG tube placement. d. Turn the patient onto his or her left side.

C

83. Which instruction might the nurse give to nursing assistive personnel (NAP) regarding the care of a patient with a nasogastric (NG) tube? a. "Remember to aspirate 5 mL to 10 mL of stomach contents before flushing the tube." b. "Let me know if the patient complains of anything related to the NG tube's placement." c. "Tell me if you see any vomit in the patient's mouth during oral care." d. "Please see if the NG tubing has advanced at all."

C

84. What would the nurse do if material aspirated from a patient's nasogastric tube resembled coffee grounds in color and texture? a. Check the tube placement. b. Assess the pH of the contents. c. Notify the health care provider. d. Irrigate the tube with water.

C

85. When checking gastric aspirate from an NG tube, the nurse assesses a pH of 7. What would the nurse do next? a. Nothing, since this is an expected pH value b. Advance the tube c. Anticipate a chest x-ray d. Pull back on the tube

C

87. What should the nurse do when a patient is ordered to receive 4 L/min oxygen by nasal cannula? a. Encourage oral fluids. b. Restrict fluids. c. Ensure that humidification is present. d. Measure blood pressure every hour

C

9. Which of the following are physiological outcomes of immobility? a. Increased metabolism b. Reduced cardiac workload c. Decreased lung expansion d. Decreased oxygen demand

C

90. What would be the nurse's priority in order to minimize a patient's risk for injury during oxygen therapy? a. Advising the patient to call for assistance before getting out of bed. b. Instructing nursing assistive personnel (NAP) to immediately correct the flow rate if the oxygen regulator is not set as prescribed. c. Observing the six rights of medication administration. d. Monitoring the patient for signs of hypoxia.

C

91. When changing the pouching system, which routine step best minimizes irritation of the skin surrounding the stoma? a. Using adhesive remover b. Emptying the ostomy bag only when full c. Avoiding unnecessary changes of the pouching system d. Wearing clean gloves

C

96. After unsuccessfully attempting to flush a nasogastric (NG) tube with water, what is the most appropriate action for the nurse to take? a. Flush the tube with ginger ale. b. Use apple juice to flush the tube. c. Obtain a product designed to unclog NG tubes. d. Force-flush the system with sterile normal saline

C

97. How could the nurse assess the patency of a nasogastric (NG) tube being used for enteral nutrition? a. Elevate the head of the patient's bed to at least 30 degrees. b. Use an intravenous fluid infusion set. c. Check the gastric residual volume. d. Monitor the amount of intake the patient tolerates in an 8-hour period.

C

98. What is the proper response to the nurse's observation that the patient's closed-system enteral feeding has 150 mL of formula remaining and that the infusion order rate is for 50 mL/hr? a. Recalculate the present drip factor for accuracy. b. Terminate the fluid, and prepare to hang a new bag of formula. c. Plan to check the feeding for completion within the next 3 hours. d. Check with the pharmacy to see if the formula has been hanging too long.

C

A new nurse is experiencing lateral violence at work. Which steps could the nurse take to address this problem? A. Challenge the nurses in a public forum to embarrass them and change their behavior B. Talk with the department secretary and ask if this has been a problem for other nurses C. Talk with the preceptor or manager and ask for assistance in handling this issue D. Say nothing and hope things get better

C

A nurse admits a 72-year-old patient with a medical history of hypertension, heart failure, renal failure, and depression to a general medical patient care unit. The nurse reviews the patient's medication orders and notes that the patient has three health care providers who have ordered a total of 13 medications. What is the most appropriate action for the nurse to take next? A. Give the medications after identifying the patient using two patient identifiers B. Provide medication education to the patient to help with adherence to the medical plan C. Review the list of medications with the health care providers to ensure that the patient needs all 13 medications D. Set up a medication schedule for the patient that is least disruptive to the expected treatment schedule in the hospital

C

A nurse in a mother-baby clinic learns that a 16year old has given birth to her first child and has not been to a well-baby class yet. The nurse.s assessment reveals that the infant cries when breastfeeding and has difficulty latching on to the nipple. The infant has not gained weight over the 2 weeks. The nurse identifies the patient's nursing diagnosis as Ineffective Breastfeeding. Which of the following is the best "related to" factor? A. Infant crying to breast B. Infant unable to latch on to breast correctly C. Mother's deficient knowledge D. Lack of infant weight gain

C

A nurse is assigned to a 42year old mother of 4 who weighs 136.2 kg (300lbs), has diabetes, and works part time in the kitchen of a restaurant. The patient is facing surgery for gallbladder disease. Which of the following approaches demonstrates the nurse's cultural competence in assessing the patient's health care problems? A. "I can tell that your eating habits have led to your diabetes. Is that right?" B. "It's been difficult for people to find jobs. Is that why you work part time?" C. "You have four children; do you have any concerns about going home and caring for them?" D. "I wish patients understood how overeating affects their health."

C

A nurse is instructing a patient who has decreased leg strength on the left side how to use a cane. Which action indicates proper cane use by the patient? A. The patient keeps the cane on the left side of the body. B. The patient slightly leans to one side while walking. C. The patient keeps two points of support on the floor at all times. D. After the patient places the cane forward, he or she then moves the right leg forward to the cane

C

A nursing assistive personnel asks for help to transfer a patient who is 125 lbs (56.8 kg) from the bed to a wheelchair. The patient is unable to help. What is the nurse's best response? A. "As long as we use proper body mechanics, no one will get hurt." B. "The patient only weighs 125 lbs. You don't need my assistance." C. "Call the lift team for additional assistance." D. "The two of us can lift the patient easily."

C

A nursing student reports to a lead charge nurse that his assigned patient seems to be less alert and his blood pressure is lower, dropping from 140/80 to 110/60. The nursing student states, "i believe this is a nursing diagnoses of Deficient Fluid Volume." The lead charge nurse immediately goes to the patient's room with the student to assess the patient's orientation, heart rate, skin turgor, and urine output for last 8 hours. The lead charge nurse suspects that the student has made which type of diagnostic error? A. Insufficient cluster of cues B. Disorganization C. Insufficient number of cues D. Evidence that another diagnosis is more likely

C

A patient has been hospitalized for the past 48 hours with a fever of unknown origin. His medical record indicates tympanic temperatures of 38.7° C (101.6° F) (0400), 36.6° C (97.9° F) (0800), 36.9° C (98.4° F) (1200), 37.6° C (99.6° F) (1600), and 38.3° C (100.9° F) (2000). How would you describe this pattern of temperature measurements? A. Usual range of circadian rhythm measurements B. Sustained fever pattern C. Intermittent fever pattern D. Resolving fever pattern

C

A patient has removed her dentures and placed them on the bedside stand. What would the nurse do to protect the patient's dentures? A. Wrap the dentures in a paper towel. B. Store the dentures in the patient's bedside stand, and notify other staff of where they have been placed. C. Obtain a denture cup, label it with the patient's name, and store the dentures in a safe place D. Wrap the dentures in a damp washcloth, and place them in a denture cup.

C

A patient is evaluated in the emergency department after causing an automobile accident while being under the influence of alcohol. While assessing the patient, which statement would be the most therapeutic? A. "Why did you drive after you had been drinking?" B. "We have multiple patients to see tonight as a result of this accident." C. "Tell me what happened before, during, and after the automobile accident tonight." D. "It will be okay. No one was seriously hurt in the accident."

C

A patient is isolated for pulmonary TB. This nurse notes that the patient seems to be angry, but he knows that this is a normal response to isolation. What is the best intervention? A. Provide a dark, quiet room to calm the patient B. Reduce the level of precautions to keep the patient from becoming angry C. Explain the reasons for isolation procedures and provide meaningful stimulation D. Limit family and other caregiver visits to reduce the risk of spreading the infection

C

A patient on prolonged bed rest is at an increased risk to develop this common complication of immobility if preventive measures are not taken: A. Myoclonus B. Pathological fractures C. Pressure ulcers D. Pruritus

C

A patient presents in the clinic with dizziness and fatigue. The nursing assistant reports a slow but regular radial pulse of 44. What is your priority intervention? A. Request that the nursing assistant repeat the pulse check B. Call for a stat electrocardiogram (ECG) C. Assess the patient's apical pulse and evidence of a pulse deficit D. Prepare to administer cardiac-stimulating medications

C

A patient receiving chemotherapy experiences stomatitis. The nurse advises the patient to use: A. Community mouthwash. B. Alcohol-based mouth rinse. C. Normal saline rinses. D. Firm toothbrush

C

A patient who is Spanish-speaking does not appear to understand the nurse's information on wound care. Which action should the nurse take? A. Arrange for a Spanish-speaking social worker to explain the procedure B. Ask a fellow Spanish-speaking patient to help explain the procedure C. Use a professional interpreter to provide wound care education in Spanish D. Ask the patient to write down questions that he or she has for the nurse

C

A patient with a history of nighttime confusion is to receive several oral medications at bedtime. What is the best way for the nurse to ensure that the patient has swallowed the medication? A. Administer each tablet individually B. Observe the patient closely as he swallows the tablets C. Ask the patient to open his mouth against swallowing each tablet D. Ask the patient to swallow a full glass of water with the tablets

C

A patient with left-sided muscle weakness is prescribed a bath every other day. Which precaution would help the nurse reduce this patient's risk of falling? A. Maintain the water temperature at 104F B. Allow the patient to remain in the bath for 45 minutes C. Decline the patient's request to add scented oil to the bathwater. D. Discuss the patient's level of fatigue after the bath.

C

As the nurse is administering medication to a patient, the patient states, "I've never seen that pill before." What is the nurse's most appropriate response? A. Reassure the patient that the pharmacy sent the right medication B. Tell the patient that it is probably a different brand that what he takes at home and not to worry C. Tell the patient that you will review the physician's order to clarify any discrepancies D. Tell the patient that the doctor probably ordered a new medication

C

During a visit to the clinic, a patient tells the nurse that he has been having headaches on and off for a week. The headaches sometimes make him feel nauseated. Which of the following responses by the nurse is an example of probing? A. So you've had headaches periodically in the last week and sometimes they cause you to feel nauseated-correct? B. Have you taken anything for your headaches? C. Tell me what makes your headaches begin D. Uh huh, tell me more

C

How can the nurse determine that the needle tip for an intradermal injection is in the dermis? A. A bleb the size of a mosquito bite will appear. B. The needle will enter at a 5- to 15-degree angle. C. The bulge of the needle tip will be visible through the skin. D. The needle will penetrate through the epidermis to a depth of about ⅛ inch.

C

How can the nurse promote infection control while providing perineal care for a female patient who has a catheter? A. By avoiding the application of tension on the catheter. B. By patting, not rubbing, the skin dry after thoroughly rinsing it. C. By cleansing the patient's labia from the pubic area toward the rectum D. By using warm water to cleanse the patient's entire perineal area.

C

How might the nurse safely administer an extended-release capsule to a patient with dysphagia? A. Encourage the patient to drink plenty of water when swallowing the capsule B. Open the capsule, the place the contents into 90 mL (3 fl. oz. ) of juice C. Place the capsule in a spoonful of the patient's applesauce D. Save the capsule to be administered last

C

How would the nurse prepare insulin to ensure its efficacy? A. Do not allow refrigerated insulin to warm up before administering it B. Follow aseptic technique during the entire process C. Roll the vial of insulin suspension between the palms prior to drawing up the medication D. Monitor the patient's blood glucose level before administering the injection

C

In which site would it be inappropriate to administer an intradermal injection? A. Lower abdomen of an obese patient B. Upper back of a patient who is on bed rest C. Right deltoid of a high school softball pitcher D. Left forearm of a patient with right-sided weakness

C

In which situation would it be appropriate for the nurse to use an antiseptic hand rub to perform hand hygiene? A. The nurse has dry cracked skin B. The nurse is sensitive to antimicrobial soap C. The nurse's hands are not visibly soiled D. The nurse has been exposed to a protein-based contaminant

C

Musculoskeletal disorders are the most prevalent and debilitating occupational health hazards for nurses. To reduce the risk for these injuries, the American Nurses Association is advocating which of the following? A. Mandate that physical therapists do all patient transfers B. Require adequate staffing levels in health care organizations C. Require the use of assistive equipment and devices D. Require an adequate number of staff to be involved in all patient transfers

C

The nurse can best determine the effect of crying on a patient's apical pulse by doing what? A. Measuring the patient's apical pulse before and after crying B. Assess the patient's apical pulse 30 minutes after crying C. Comparing the patient's post-crying apical pulse with her baseline or previous rate D. Measuring the patient's pulse deficit after crying

C

The nurse has one bed alarm available and can use it for any of the following patients, all of whom have dementia. Having an alarm is most important for which patient A. A patient who has refused most meals for the past week and whose weight has dropped by 10% in the past month B. A patient who has become verbally combative with staff in recent weeks C. A patient who was returned to the unit last week by staff in an adjacent assisted living facility D. A patient whose abdominal feeding tube is covered with an abdominal binder

C

The nurse is bathing a patient who is unconscious. What should the nurse do to ensure safe care of the patient's eyes? A. Remove eye crusts with soapy water. B. Avoid closing the patient's eyes. C. Use eye patches or shields taped in place. D. Tape the patient's eyelids closed.

C

The nurse is helping a patient get out of a bathtub, and the patient appears to be unsteady on her feet. What should the nurse do to help ensure the patient's safety? A. Drape a bath towel over the patient's shoulders B. Demonstrate how to use the call light for assistance C. Drain the bathtub before the patient gets out D. Apply lotion to the patient's freshly dried skin

C

The nurse is preparing to apply a topical oil-based medication to a patient's forearms. What should the nurse do to minimize the risk of contamination during the application? A. Encourage the patient to self-apply the medication B. Wear treatment gloves during the entire application process C. Change gloves between prepping the skin and applying the medication D. Perform effective hand hygiene before and after applying the medication

C

The nurse is preparing to discharge a patient after instructing her how to apply her own estrogen patch. What is the best way for the nurse to follow up? A. Tell the patient what the most common errors in application are. B. Review the material with the patient and family C. Evaluate the patient's ability to apply the patch D. Give the patient printed materials for later reference

C

The nurse is preparing to discharge a patient after instructing her in self-application of a topical medication. What is the best way for the nurse to ensure that the patient understands the instructions? A. Discuss with the patient the most common errors in application B. Review the material several times with the patient and family C. Allow the patient to apply the topical medication and provide feedback on technique D. Give the patient printed materials for later reference

C

The nurse observes a patient walking down the hall with a shuffling gait. When the patient returns to bed, the nurse checks the strength in both of the patient's legs. The nurse applies the information gained to suspect that the patient has a mobility problem. This conclusion is an example of: A. Cue B. Reflection C. Clinical Inference D. Probing

C

The nurse observes an adult Middle Eastern patient attempting to bathe himself with only his left hand. The nurse recognizes that this behavior likely relates to: A. Obsessive compulsive behavior. B. Personal preferences. C. The patient's cultural norm. D. Controlling behaviors

C

The nurse observes the NAP providing perineal care to a male patient. Which observation of care requires the nurse's follow-up? A. Assisting the patient into a supine position in bed. B. Cleansing the tip of the penis with a circular motion, starting at the meatus. C. Reserving the cleansing of the tip of the penis as the final step in perineal care. D. Using a gloved hand to grasp the shaft of the penis in order to retract the foreskin

C

The nurse plans to assess a patient's respiratory rate; however, the patient has just returned from ambulating to the bathroom. What should the nurse do to minimize the effect of exercise on the patient's respiratory rate? A. Assess the pulse for a full 60 seconds before assessing respiration B. Compare the postexercise respiratory rate with his baseline findings C. Encourage the patient to rest for 10 minutes before assessing respiration D. Compare the postexercise findings with the previous at-rest findings

C

The nurse who is preparing to make an unoccupied bed should do what to ensure his or her personal safety? A. Put on sterile gloves. B. Place the call light within the nurse's reach C. Place the bed at a comfortable working height D. Place a laundry bag on the bedside chair.

C

The nursing assistive personnel (NAP) informs you that the electronic blood pressure machine on the patient who has recently returned from surgery following removal of her gallbladder is flashing a blood pressure of 65/46 and alarming. What is the correct order for care activities? 1. Press the start button of the electronic blood pressure machine to obtain a new reading. 2. Obtain a manual blood pressure with a stethoscope. 3. Check the patient's pulse distal to the blood pressure cuff. 4. Assess the patient's mental status. 5. Remind the patient not to bend her arm with the blood pressure cuff. A. 5, 3, 1, 4, 2 B. 3, 2, 1, 4, 5 C. 4, 1, 3, 2, 5 D. 1, 2, 4, 3, 5 E. 2, 3, 1, 4, 5

C

What can the nurse do to keep the patient from becoming chilled while receiving a bath with a disposable bath-in-a-bag product? A. Dry each body part with a warmed towel after washing. B. Wash the product off of the skin with a warm, moistened washcloth. C. Lightly cover the patient with a bath towel. D. Keep the patient's gown on for the bath

C

What can the nurse do to minimize the discomfort of a subcutaneous injection? A. Inject the medication rapidly B. Massage the injection site C. Cover the injection site with gauze pad after withdrawing the needle D. Inject the medication without pinching the skin

C

What can the nurse do to minimize the patient's risk for injury when delivering an intramuscular injection? A. Instruct the patient to relax B. Insert the needle at a 45-degree angle C. Pull back on the plunger after inserting the needle D. Pull the skin taut at the injection site when inserting the needle

C

What is the correct order in which elastic stockings should be applied? 1. Identify patient using two identifiers. 2. Smooth any creases or wrinkles. 3. Slide the remainder of the stocking over the patient's heel and up the leg 4. Turn the stocking inside out until heel is reached. 5. Assess the condition of the patient's skin and circulation of the legs. 6. Place toes into foot of the stocking. 7. Use tape measure to measure patient's legs to determine proper stocking size. A. 1, 5, 7, 4, 6, 2, 3 B. 1, 7, 5, 4, 6, 2, 3 C. 1, 5, 7, 4, 6, 3, 2 D. 1, 5, 4, 7, 6, 3, 2

C

What is the greatest safety concern when withdrawing medication from an ampule? A.Not wearing gloves when preparing medication B. Selecting an inappropriate needle size C. Withdrawing glass particles into the syringe D. Withdrawing bubbles into the syringe

C

What is the most effective way to control transmission of infection? A. Isolation precautions B. Identifying the infectious agent C. Hand hygiene practices D. Vaccinations

C

What is the most important step the nurse can take to ensure that the patient is getting the correct medication? A. Assess the patient's ability to swallow oral medications without difficulty B. Question the patient about his or her experience with this or similar medications C. Compare the medication label with the MAR three times. D. Evaluate the patient's understanding of the safety issues related to the specific drug

C

What is the primary reason the nurse encourages the patient to participate in hair care? A. To free up the staff's time for patient care B. To make sure the care is performed according to the patient's performance C. To encourage the patient's sense of independence D. To allow the nurse to evaluate the patient's ability to manipulate objects

C

What should the nurse do if the patient's blood pressure is not within normal limits? A. Review the blood pressure readings in the patient's record B. Assess for proper cuff size and arm positioning C. Promptly report the assessment data to the nurse in charge or to the health care provider D. Encourage the patient to rest quietly in bed for 30 minutes, and then retake the blood pressure

C

What will the nurse do when a gown change is needed for a patient who is receiving intravenous fluids delivered by a pump? A. Document that the tubing had to be disconnected in order to change the gown B. Ask another nurse to monitor the pump during the gown change C. Pause the infusion by pressing the sensor on the pump D. Help the patient maintain good hygiene until the infusion is discontinued and the gown can be changed

C

What would the nurse do to prevent the spread of infection when assessing a patient's blood pressure? A. Wear gloves B. Avoid using an arm in which an intravenous catheter has been inserted C. Clean the stethoscope with alcohol before and after using it D. Inflate the cuff 30 mm higher than the expected systolic pressure

C

When brushing the teeth of an unconscious patient, why is the toothbrush held so that its bristles are at a 45-degree angle to the gum line? A. To give the nurse a firm grip on the brush handle B. To ensure that the bristles reach all tooth surfaces C. To allow the bristles to reach beneath the gum line D. To reduce pressure on sensitive oral tissues

C

When caring for a patient with Alzheimer's disease, why does the nurse cover the external urinary collection catheter? A. To protect the bed from being soiled B. To avoid offending visitors who would otherwise see the device C. To reduce the patient's access to the device D. To keep the patient from trying to get out of bed alone

C

When delegating patient care that requires NAP to use PPE, it is necessary for the nurse to do what first? A. Discuss what equipment and supplies to bring to the patient's room B. Document that the care was delegated to the NAP C. Review the patient's need for a specific isolation precaution D. Observe the NAP donning the appropriate PPE

C

When instructing a patient in the use of a dry powder inhaler, which statement is accurate? A. It is important to shake the DPI before administering the medication B. It is important to exhale while the lips are still around mouthpiece C. It is important to read the manufacturer's instructions to determine how quickly to inhale the medication D. It is important for the patient to hold his or her breath for at least 60 seconds after inhaling the medication

C

When make an unoccupied bed, where would the nurse place a waterproof pad? A. Directly on the mattress B. Beneath the drawsheet C. Over the bottom sheet. D. Over the top sheet

C

When preparing an injection from an ampule, what will the nurse do if liquid is trapped in the neck of the ampule? A.Check the medication cabinet for an extra ampule of the medication B. Notify the pharmacy that an additional ampule of medication will be needed C. Use quick, light finger taps on the top of the ampule to move the liquid D. Shake the medication out of the neck of the ampule

C

When preparing an intramuscular injection, what can the nurse do to reduce the patient's risk for infection? A. Wear clean gloves B. Use a 3-mL syringe C. Clean the injection site with an alcohol swab D. Massage the injection site

C

When removing a gown worn as PPE while caring for a patient in isolation, why does the nurse avoid touching the outside of the gown? A. To ensure that the gown can be reused B. To protect the nurse's uniform C. To prevent touching contaminated material with unprotected hands D. To fold the gown correctly for reuse by the same nurse

C

Which action ensures that a patient will not have unnecessary pain during a linen change? A. Discontinue the bed change if the patient expresses or displays physical signs of pain B. Explain the procedure to the patient before beginning the linen change C. Administer a prescribed analgesic 30 to 60 minutes before the bed change if needed. D. Postpone the bed change if the patient reports having physical pain before you begin

C

Which action minimizes the risk of introducing glass particles into the syringe when drawing medication from an ampule? A.Using minimal force to snap the neck of the ampule B.Using gauze to cover the top of the ampule when snapping it C.Using a filter needle or straw to draw the medication from the ampule D.Allowing the medication to settle after the ampule has been snapped open

C

Which action would be avoided when providing hair care for a bed-bound patient with a history of dizziness? A. Raising the patient quickly into a sitting position after completing a bed shampoo B. Getting water into a patient's ears during the rinsing phase of the shampoo C. Placing the neck in a hyperextended position during the shampoo process D. Having the entire shampooing process last longer than 15 minutes

C

Which action would the nurse take when mixing intermediate- and long-acting insulins together in one syringe? A. Draw the intermediate-acting insulin into the syringe first B. Draw the long-acting insulin into the syringe first C. Prepare two injections D. Draw either the intermediate- or the long-acting insulin into the syringe first

C

Which finding tells the nurse that a patient may have had a positive reaction to a tuberculin test? A. A raised wheal the size of a mosquito bite B. A bruised area 10 mm or greater in diameter C. A hard, raised area 15 mm or greater in diameter D. A flat, reddened area 5 mm or greater in diameter

C

Which instruction might the nurse give to NAP that is applicable only to temporal artery temperature assessment? A. An accurate temperature reading is obtained with moisture on the forehead B. Put on a disposable sensor cover before taking the temporal artery temperature C. Place the sensor flush on the patient's forehead D. Obtain the temperature reading on the lower neck

C

Which needle would be most appropriate for the nurse to use when giving a subcutaneous injection to a patient of average height and weight? A. 20-gauge, ½-inch B. 22-gauge, 1-inch C. 25-gauge, ⅜-inch D. 27-gauge, 1 inch

C

Which nursing action best evaluates the effectiveness of an antipyretic medication in a patient with an oral temperature of 101.6F? A. Assess for physical aches B. Assess skin temperature by touching the forehead C. Assess oral temperature 30 minutes after the agent is administered D. Assess skin color for signs of fever-related flushing

C

Which of the following are physiological outcomes of immobility? A. Increased metabolism B. Reduced cardiac workload C. Decreased lung expansion E. Decreased oxygen demand

C

Which of the following indicates that additional assistance is needed to transfer the patient from the bed to the stretcher? A. The patient is 5 feet 6 inches and weighs 120 lbs. B. The patient speaks and understands English. C. The patient received an injection of morphine 30 minutes ago for pain. D. You feel comfortable handling a patient of this size and level of cooperation.

C

Which of the following nursing actions addresses the risk for infection related to fecal occult blood testing? A. Maintaining aseptic technique while performing the test B. Performing the fecal occult blood testing in the patient's bathroom C. Wearing clean gloves while testing D. Assessing the patient's ability to provide an uncontaminated fecal specimen

C

Which of the following statements is accurate regarding insulin administration? A. Vials of insulin may be stored in the freezer to extend their shelf life. B. If the rapid-acting insulin ordered is unavailable, it is safe to substitute an alternative rapid-acting insulin C. Vials of insulin must be inspected before each use for changes in appearance D. All insulin must be shaken before use to redistribute particles within the suspension

C

Which patient is at highest risk for tachycardia? A. A healthy basketball player during warmup exercises B. A patient admitted with hypothermia C. A patient with a fever of 39.4° C (103F) D. A 90-year-old male taking beta blockers

C

Which precaution should the nurse observe to ensure patient safety when using an automated medication dispensing system? A. Assess the patient's ability to swallow oral medications without difficulty. B. Ask the patient about his or her experience with this or similar medications. C. Prepare medications for one patient at a time. D. Evaluate the patient's understanding of the safety issues related to the specific drug.

C

Which statement best illustrates the nurse's understanding of the role of NAP in applying an estrogen patch? A. "Let me know when it's time to change the patient's patch" B. "Take care not to apply the patch over breast tissue." C. "Please apply lotion to the site from which the old patch was removed." D. "Make a note of where the patch s now before you remove it."

C

Which statement might the nurse make to nursing assistive personnel (NAP) when caring for a patient who is prescribed an intradermal injection? A. "Be sure to wear clean gloves during the injection." B. "Tell him it's OK; the site should look like a mosquito bite." C. "Immediately report any patient complaints of itching or dyspnea." D. "Remind the patient to come back in 48 to 72 hours so we can evaluate the site."

C

While planning morning care, which of the following patients would have the highest priority to receive his or her bath first? A. A patient who just returned to the nursing unit from a diagnostic test B. A patient who prefers a bath in the evening when his wife visits and can help him C. A patient who is experiencing frequent incontinent diarrheal stools and urine D. A patient who has been awake all night because of pain 8/10

C

While preparing a patient's oral medication dispensed from an automated medication dispensing system, the nurse realizes that the pill dispensed is twice the correct dose. What is the nurse's best action at this time? A. Notify the health care provider and ask if the higher dose could be given. B. Access the dispenser again for the correct dose. C. Notify the pharmacy to determine if the accurate dose is available. D. Split the pill in half.

C

Why does the nurse instruct NAP to remove the wrist restraint of a confused patient every 2 hours? A. To try a less restrictive type of restraint if a more confining restraint has proved effective B. To double-check the size by inserting one finger between the wrist and the restraint C. To check the skin integrity and range of motion of the wrist D. To comply with Joint Commission standards

C

Why would the nurse plan to perform foot care for a patient with PVD, rather than delegate this activity to NAP? A. The patient prefers that the nurse provide the care B. NAP are not trained to perform foot care C. The patients elevated risk of infection makes it unsafe for NAP to perform care D. The patients condition requires that he remain on bed rest.

C

You are caring for a patient who frequently tries to remove his intravenous catheter and feeding tube. You have an order from the health care provider to apply a wrist restraint. What is the correct order for applying a wrist restraint? 1. Be sure that patient is comfortable with arm in anatomic alignment. 2. Wrap wrist with soft part of restraint toward skin and secure snugly. 3. Identify patient using two identifiers. 4. Introduce self and ask patient about his feelings of being restrained. 5. Assess condition of skin where restraint will be placed. A. 4, 3, 5, 1, 2 B. 4, 3, 1, 5, 2 C. 3, 4, 1, 5, 2 D. 3, 4, 5, 1, 2

C

Integrity of the oral mucosa depends on salivary secretion. Which of the following factors impairs salivary secretion? (Select all that apply.) A. Use of cough drops B. Immunosuppression C. Radiation therapy D. Dehydration E. Presence of oral airway

C, D

When you are assigned to a patient who has a reduced level of consciousness and requires mouth care, which physical assessment techniques should you perform before the procedure? (Select all that apply.) A. Oxygen saturation B. Heart rate C. Respirations D. Gag reflex E. Response to painful stimulus

C, D

You are admitting Mr. Jones, a 64-year-old patient who had a right hemisphere stroke and a recent fall. His wife stated that he has a history of high blood pressure, which is controlled by an antihypertensive and a diuretic. Currently he exhibits left-sided neglect and problems with spatial and perceptual abilities and is impulsive. He has moderate left-sided weakness that requires the assistance of two and the use of a gait belt to transfer to a chair. He currently has an intravenous (IV) line and a urinary catheter in place. Which factors increase his fall risk at this time? (Select all that apply.) A. Smokes a pack a day B. Used a cane to walk at home C. Takes antihypertensive and diuretics D. History of recent fall E. Neglect, spatial and perceptual abilities, impulsive F. Requires assistance with activity, unsteady gait G. IV line, urinary catheter

C, D, E, F, G

The family of a patient who is confused and ambulatory insists that all four side rails be up when the patient is alone. What is the best action to take in this situation? (Select all that apply.) A. Contact the nursing supervisor. B. Restrict the family's visiting privileges. C. Ask the family to stay with the patient if possible. D. Inform the family of the risks associated with side-rail use. E. Thank the family for being conscientious and put the four rails up. F. Discuss alternatives that are appropriate for this patient with the family.

C, D, F

Which of the following is a principle of proper body mechanics when lifting or carrying objects? (Select all that apply.) A. Keep the knees in a locked position. B. Bend at the waist to maintain a center of gravity. C. Maintain a wide base of support. D. Hold objects away from the body for improved leverage. E. Encourage patient to help as much as possible.

C, E

27. Which of the following is a principle of proper body mechanics when lifting or carrying objects? (Select all that apply.) a. Keep the knees in a locked position. b. Bend at the waist to maintain a center of gravity. c. Maintain a wide base of support. d. Hold objects away from the body for improved leverage. e. Encourage patient to help as much as possible.

C,E

As you are obtaining the oxygen saturation on a 19-year-old college student with severe asthma, you note that she has black nail polish on her nails. You remove the polish from one nail, and she asks you why her nail polish had to be removed. What is the best response? A. Nail polish attracts microorganisms and contaminates the finger sensor. B. Nail polish increases oxygen saturation. C. Nail polish interferes with sensor function. D. Nail polish creates excessive heat in sensor probe.

C.

The nurse observes a nursing student taking a blood pressure (BP) on a patient. The nurse notes that the student very slowly deflates the cuff in an attempt to hear the sounds. The patient's BP range over the past 24 hours is 132/64 to 126/72 mm Hg. Which of the following BP readings made by the student is most likely caused by an incorrect technique? A. 96/40 mm Hg B. 110/66 mm Hg C. 130/90 mm Hg D. 156/82 mm Hg

C.

Enteral nutrition is indicated for what condition? Small bowel resection Intestinal obstruction Extended bowel rest Cerebrovascular accident

Cerebrovascular accident

Cleansing enemas help with _____________. Dyspepsia Flatulence Increase appetite Complete evacuation of feces

Complete evacuation of feces

105. What would the nurse do first when preparing to educate the patient about safe administration of oxygen therapy at home? a. Evaluate the patient's understanding of the combustible nature of oxygen. b. Arrange for a capable family member to be present during the initial discussion. c. Collect written information to present to the patient as supplemental instructional materials. d. Assess the patient's emotional readiness and physical ability to provide autonomous care.

D

108. During an admission interview, a patient who is required to stay in the supine position reports, "I can't breathe well while I'm lying down." What would the nurse do first to help this patient? a. Notify the health care provider of the patient's complaint. b. Request that the health care provider prescribe oxygen therapy. c. Interview the patient concerning the onset of this problem. d. Instruct the patient to use two bed pillows when lying supine

D

12. The nurse is preparing to logroll a patient in bed. Why are two assistants needed on the side toward which the patient is being turned a. To position the pillows b. To ease the patient back onto the support pillows c. To keep the spine in alignment d. To roll the patient as a unit

D

17. The nurse is preparing to transfer a patient with left-sided weakness from the bed to a wheelchair using a transfer belt. Which positioning would the nurse instruct the patient to assume a. Place both feet together on the floor b. Place your weaker foot forward and your stronger leg toward the back. c. Extend bot of your legs and feet d. Place your stronger leg forwards and your weaker leg toward the back.

D

20. Which instruction would the nurse give a patient who is able to assist with transfer from a bed to a wheelchair using a transfer belt? a. "When I count to three, please rock yourself into a standing position" b. "Please hold on to my waist while I help you stand." c. "Please tell me how I can best help you get up off the bed and stand up." d. "Please push down onto the mattress with both hands and stand when I count to three"

D

24. The nurse encourages a patient with type 2 diabetes to engage in a regular exercise program primarily to improve the patient's: a. Gastric motility, thereby facilitating glucose digestion. b. Respiratory effort, thereby decreasing activity intolerance. c. Overall cardiac output, thereby resuming resting heart rate d. Use of glucose and fatty acids, thereby decreasing blood glucose level.

D

28. The nurse is preparing to delegate the application of a SCD to nursing assistive personnel (NAP). Which statement by the NAP requires follow-up by the nurse? a. "I will check for a green light on the mechanical unit" b. "I will remove the SCD before ambulating the patient c. "I will tell you if I see any signs of itching, redness, or irritation on the patients legs" d. "I will measure the patient's legs to determine what size SCD sleeve to use"

D

29. Which of the following diagnosis is a patient who started smoking in adolescence and continues to smoke for 40 years at this risk for? a. Alcoholism and hypertension b. Obesity and diabetes c. Stress-related illnesses d. Cardiopulmonary disease and lung cancer

D

3. An older-adult patient has been bedridden for 2 weeks. Which of the following complaints by the patient indicates to the nurse that he or she is developing a complication of immobility? a. Loss of appetite b. Gum soreness c. Difficulty swallowing d. Left-ankle joint stiffness

D

30. A patient has been diagnosed with severe iron deficiency anemia. During physical assessment, which of the following symptoms are associated with decreased oxygenation? a. Increased breathlessness but increased activity tolerance b. Decreased breathlessness and decreased activity tolerance c. Increased activity tolerance and decreased breathlessness d. Decreased activity tolerance and increased breathlessness

D

33. A patient is admitted with the diagnosis of severe left-sided heart failure. What adventitious lung sounds are expected on auscultation? a. Sonorous wheezes in the left lower lung b. Rhonchi mid sternum c. Crackles only in apex of lungs d. Inspiratory crackles in lung bases

D

35. Why does the nurse remove the patient's elastic stockings at least once per shift? a. To permit the skin to breathe b. To wash the legs with a disposable bath product c. To air out the stockings and allow sweat to evaporate d. To check the skin for irritation or breakdown

D

36. A patient has been newly diagnosed with chronic lung disease. In discussing his condition with the nurse, which of his statements would indicate a need for further education? a. "I'll make sure that I rest between activities so I don't get so short of breath." b. "I'll practice the pursed-lip breathing technique to improve my exercise tolerance." c. "If I have trouble breathing at night, I'll use two to three pillows to prop up." d. "If I get short of breath, I'll turn up my oxygen level to 6 L/min."

D

36. The nurse is performing passive shoulder and elbow exercises for a patient who is revering from surgery to remove a soft-tissue tumor in her upper arm. Why does the nurse cup one hand around the patient's elbow and support the forearm and wrist during the ROM exercise? a. To keep the arm above the level of the heart b. To assess the patient's muscle tension c. To listen for crepitus in the joint d. To ensure stability while exercising the joint

D

39. The nurse is reviewing the results of the patient's diagnostic testing. Of the following results, the finding that falls within expected or normal limits is: a. Palpable, elevated hardened area around a tuberculosis skin testing site. b. Sputum for culture and sensitivity identifies mycobacterium tuberculosis c. Presence of acid fast bacilli in sputum d. Arterial oxygen tension (PaO2) of 95 mmHg

D

44. The nurse evaluates which laboratory values to assess a patient's potential for wound healing? a. Fluid status b. Potassium c. Lipids d. Nitrogen balance

D

45. The nurse has delegated administration of a standard enema for a 72-year-old patient with constipation. Which statement made by nursing assistive personnel (NAP) requires the nurse to follow-up? a. "I'll warm up the solution before instilling it." b. "I'll place the patient in the left side-lying position with the right knee bent." c. "I'll put a waterproof pad under the patient before I start." d. "I'll instill the solution and then check in on my other patients until I get the call signal."

D

47. A patient is receiving both parenteral (PN) and enteral nutrition (EN). When would the nurse collaborate with the health care provider and request discontinuing parenteral nutrition? a. When 25% of the patient's nutritional needs are met by the tube feedings b. When bowel sounds return c. When central line has been in for 10 days d. When 75% of the patient's nutritional needs are met by the tube feedings

D

49. A patient's gastric residual volume was 250 mL at 0800 and 350 mL at 1200. What is the appropriate nursing action? a. Assess bowel sounds b. Raise the head of the bed to at least 45 degrees c. Position the patient on his or her right side to promote stomach emptying d. Do not reinstall aspirate and hold the feeding until you talk to the primary care provider

D

49. The nurse is assisting with a bedpan for a patient who had knee surgery 24 hours ago. What is the best way for the nurse to maximize comfort while the patient uses the bedpan? a. Raise the knee gatch b. Offer a dose of the patient's prescribed oral pain medication c. Evaluate the patient's ability to move in bed. d. Elevate the head of the bed to between 30 and 60 degrees

D

51. Which statement made by a patient of a 2-month-old infant requires further education? a. I'll continue to use formula for the baby until he is a least a year old. b. I'll make sure that I purchase iron-fortified formula. c. I'll start feeding the baby cereal at 4 months. d. I'm going to alternate formula with whole milk starting next month.

D

57. Which nursing action addresses the risk for infection related to gastric occult blood testing? a. Maintaining aseptic techniques while handling the Gastroccult slide b. Perform the test in the patient's bathroom c. Assessing the patient's history of previous gastrointestinal (GI) bleeding d. Ensuring appropriate hand hygiene before and after testing

D

64. During the nursing assessment a patient reveals that he has diarrhea and cramping every time he has ice cream. He attributes this to the cold nature of the food. However, the nurse begins to suspect that these symptoms are associated with what problem? a. Food allergy b. Irritable bowel c. Increased peristalsis d. Lactose intolerance

D

68. What is the correct order for an ostomy pouch change? 1. Close the end of the pouch. 2. Measure the stoma. 3. Cut the hole in the wafer. 4. Press the pouch in place over the stoma. 5. Remove the old pouch. 6. Trace the correct measurement onto the back of the wafer. 7. Assess the stoma and the skin around it. 8. Cleanse and dry the peristomal skin. a. 5, 8, 2, 7, 3, 6, 4, 1 b. 8, 5, 6, 2, 7, 3, 4, 1 c. 8, 5, 7, 6, 2, 3, 4, 1 d. 5, 8, 7, 2, 6, 3, 4, 1

D

69. The nurse is preparing to administer a rectal suppository to an elderly patient. Which step best protects the patient's safety? a. Ask the patient to take deep, slow breaths as the suppository is being inserted. b. Insert the suppository 2 inches into the rectum. c. Place the patient in the left side-lying position with the top leg flexed. d. Instruct the patient to use the call light for assistance to the bathroom

D

7. Which of the following nursing interventions should be implemented to maintain a patent airway in a patient on bed rest? a. Isometric exercises b. Administration of low-dose heparin c. Suctioning every 4 hours d. Use of incentive spirometer every 2 hours while awake

D

9. A patient will be moved in bed with the use of a fracture-reducing device. How will the nurse place this device under the patient? a. Lift the patient to place the device directly under him or her b. Remove the drawsheet, and replace it with the device c. Sit the patient up in the bed, and place the device behind the shoulders. d. Roll the patient from side to side, and place the device under the drawsheet

D

92. Which initial nursing action would best help the patient learn self-care of a colostomy pouching system? a. Giving the patient handouts on self care of a colostomy b. Allowing the patient to examine an ostomy device c. Identifying a family member who can participate in the ostomy appliance process d. Giving the patient a mirror to watch the nurse provide care

D

95. When pouching a patient's colostomy, which action reduces the patient's risk for injury? a. Measuring output when emptying the contents of the pouch b. Maintaining the patient's bowel elimination function c. Promoting the patient's autonomy with bowel elimination care d. Protecting the skin from irritation caused by fecal drainage

D

A 52-year-old woman is admitted with dyspnea and discomfort in her left chest with deep breaths. She has smoked for 35 years and recently lost over 10 lbs. Her vital signs on admission are: HR 112, BP 138/82, RR 22, tympanic temperature 36.8° C (98.2° F), and oxygen saturation 94%. She is receiving oxygen at 2 L via a nasal cannula. Which vital sign reflects a positive outcome of the oxygen therapy? A. Temperature: 37° C (98.6° F) B. Radial pulse: 112 C. Respiratory rate: 24 D. Oxygen saturation: 96% E. Blood pressure: 134/78

D

A 72 year old male patient comes to the health clinic for an annual follow-up. The nurse enters the patient's room and notices him to be diaphoretic, holding his chest and breathing with difficulty. The nurse immediately checks the patient's heart rate and blood pressure and asks him, "tell me where your pain is." Which of the following assessment approaches does this scenario describe? A. Review of systems approach B. Use of a structured database format C. Back channeling D. A problem-oriented approach

D

A family member is providing care to a loved on who has an infected leg wound. What should the nurse instruct the family member to do after providing care and handling contaminated equipment or organic material? A. Wear gloves before eating or handing food B. Place any soiled materials into a bag and double bag it C. Have the family member check with the health care provider about need for immunizations D. Perform hand hygiene after care and/or handling contaminated equipment or material.

D

A nurse accidently gives a patient the medications that were ordered for the patient's roommate. What is the nurse's first priority? A. Complete an occurrence report B. Notify the health care provider C. Inform the charge nurse of the error D. Assess the patient for adverse effects

D

A nurse assess a young woman who works part time but also cares for her mother at home. The nurse reviews clusters of data that include the patient's report of frequent awakenings at night, reduced ability to think clearly at work, and a sense of not feeling well rested. Which of the following diagnoses is in the correct PES format? A. Disturbed Sleep Pattern evidenced by frequent awakening B. Disturbed Sleep Pattern related to family caregiving responsibilities C. Disturbed Sleep Pattern related to need to improve sleep habits D. Disturbed Sleep Pattern related to caregiving responsibilities as a evidenced by frequent awakening and not feeling rested

D

A nurse assesses a patient who comes to the pulmonary clinic. "I see that it's been over 6 months since you've been here, but your appointment was for every 2 months. Tell me about that. Also I see from your last visit that the doctor recommended routine exercise. Can you tell me how successful you've been in following his plan?" The nurse's assessment covers which of Gordon's functional health patterns? A. Value-belief pattern B. Cognitive-perceptual pattern C. Coping-stress-tolerance pattern D. Health perception-health management pattern

D

A nurse interviewed and conducted a physical examination of a patient. Among the assessment data the nurse gathered were an increased respiratory rate, the patient reporting difficulty breathing while lying flat, and pursed-lip breathing. This data set is an example of: A. Collaborative data set B. Diagnostic label C. Related factors D. Data cluster

D

A nurse is assigned to care for a patient for the first time and states, "I don't know a lot about your culture and want to learn how to better meet your health care needs." Which therapeutic communication technique did the nurse use in this situation? A. Validation B. Empathy C. Sarcasm D. Humility

D

A nurse is assigned to care for the following patients. Which of the patients is most at risk for developing skin problems and thus requiring thorough bathing and skin care? A. A 44-year-old female who has had removal of a breast lesion and is having her menstrual period. B. A 56-year-old male patient who is homeless and admitted to the emergency department with malnutrition and dehydration and who has an intravenous line. C. A 60-year-old female who experienced a stroke with right-sided paralysis and has an orthopedic brace applied to the left leg. D. A 70-year-old patient who has diabetes and dementia and has been incontinent of stool

D

A nurse is reviewing a patient's list of nursing diagnosis in the medical record. The most recent nursing diagnosis is Diarrhea related to intestinal colitis. For which of the following reasons is this an INCORRECTLY stated diagnostic statement? A. Identifying the clinical sign instead of an etiology B. Identifying a diagnosis on the basis of prejudicial judgement C. Identifying the diagnostic study rather than a problem caused by the diagnostic study D. Identifying the medical diagnosis instead of the patient's response to the diagnosis

D

A nursing student is administering medications to a patient through a gastric tube (G-tube). Which of the following actions taken by the nursing student requires the nursing instructor to intervene? A. The nursing student places all the patient's medications in different medicine cups. B. The nursing student evaluates each medication and holds the tube feeding before administering a medication that needs to be administered on an empty stomach C. The nursing student flushes the tube with 30 mL of water between each medication D. The nursing student crushes a nifedipine extended-release tablet and mixes it with water before administering it.

D

A nursing student is reviewing a process recording with the instructor. The student engaged the patient in a discussion about availability of family members to provide support at home once the patient is discharged. The student reviews with the instructor whether the comments used encouraged openness and allowed the patient to "tell his story." This is an example of which step of the nursing process? A. Planning B. Assessment C. Intervention D. Evaluation

D

A parent calls the pediatrician's office to ask about directions for using a car seat. Which of the following is the most correct set of instructions the nurse gives to this parent? A. Only infants and toddlers need to ride in the back seat B. All toddlers can move to a forward facing car seat when they reach age 2. C. Toddlers must reach age 2 and the height/weight requirement before they ride forward facing. D. Toddlers must reach age 2 or the height or weight requirement before they ride forward facing.

D

A patient has been admitted for a cerebrovascular accident (stroke). She cannot move her right arm, and she has a right-sided facial droop. She is able to eat with her dentures in place and swallow safely. The nursing assistive personnel (NAP) reports to you that the patient will not keep the oral thermometer probe in her mouth. What direction do you provide to the NAP? A. Direct the NAP to hold the thermometer in place with her gloved hand B. Direct the NAP to switch the thermometer probe to the left sublingual pocket C. Direct the NAP to obtain a right tympanic temperature D. Direct the NAP to use a temporal artery thermometer from right to left

D

A patient has been prescribed a metered-dose inhaler (MDI) containing 200 doses of a bronchodilator. The patient has been instructed to take two puffs of the medication three times daily. At this dosage, how long will the MDI last? A. 100 days B. 50 days C. 66 days D. 33 days

D

A patient has consumed three 100-mL cups of ice chips and 4 ounces of ginger ale. What will nursing assistive personnel (NAP) document as this patient's oral intake? a. 120 mL b. 170 mL c. 220 mL d. 270 mL

D

A patient is being given a bed bath. The nurse realizes that another washcloth is needed to complete the bath. What is one way in which the nurse can ensure the patient's safety? A. Use the call light to ask someone else to bring a washcloth. B. Raise all four side rails on the patient's bed. C. Raise the bed to its highest position. D. Make sure the call light is within the patient's reach.

D

A patient tells the nurse that at home he cleans his dentures after every meal and before going to bed. When would denture care be planned for this patient while hospitalized? A. After breakfast and before going to bed B. With morning care C. With morning and evening care D. After every meal and before going to bed

D

A patient who visits the surgery clinic 4 weeks after a traumatic amputation of his right leg tells the nurse practitioner that he is worried about his ability to continue to support his family. He tells the nurse he feels that he has let his family down after having an auto accident of his right leg. The nurse listens and then asks the patient, "How do you see yourself now?" On the basis of Gordon's functional health patterns, which pattern does the nurse assess? A. Health perception-health management pattern B. Value-belief pattern C. Cognitive-perceptual pattern D. Self-perception-self-concept pattern

D

A patient with diabetes remarks during foot care that she has been letting her skin air-dry after bathing at home because her doctor told her to use plenty of moisturizer on her hands and feet. What should the nurse teach the patient? A. To apply moisturizer after air-drying thoroughly B. To apply moisturizer while the skin is still wet C. To skip the moisturizer D. To towel-dry thoroughly before applying moisturizer

D

A toddler is to receive 2.5 mL of an antipyretic by mouth. Which equipment is the most appropriate for medication administration for this child? A. A medication cup B. A teaspoon C. A 5-mL syringe D. An oral-dosing syringe

D

After bacteria are cultured from a midstream urine specimen, what is accomplished by sensitivity testing? A. Confirms the accuracy of the results of the culture B. Identifies the immune system's reaction to the presence of the bacteria C. Determines whether the patient is allergic to the antibiotic agent with which the provider plans to treat the infection D. Determines which antibiotic agent is most effective in killing the bacteria

D

An older-adult patient has been bedridden for 2 weeks. Which of the following complaints by the patient indicates to the nurse that he or she is developing a complication of immobility? A. Loss of appetite B. Gum soreness C. Difficulty swallowing D. Left-ankle joint stiffness

D

As the nurse is giving a patient his medications, he remarks, "I've never seen this blue pill before." What is the nurse's correct response? A. "I'm sure the doctor knows what he's doing. Don't worry." B. "Our pharmacy probably sent a generic form of what you're used to taking." C. "What color pill are you used to seeing?" D. "Don't take it. Let me double-check the doctor's order to make sure this is the correct medication for you."

D

As the nurse prepares to administer oral acetaminophen, the patient refuses to accept the drug because it doesn't look like the Tylenol she takes at home. After verifying that the medication and dosage are correct, what is the nurse's best response? A. Inform the patient that the medication is a form of Tylenol B. Explain that she will probably have increased pain if she refuses the medication C. Show the patient a picture of the medication D. Explain that drugs often come in different physical forms, depending on the manufacturer

D

During the assessment of a patient's respiratory rate, when the second hand reaches the 15-second mark, the respiratory count is 8. What should the nurse do at this time? A. Stop the assessment B. Stop the assessment, and multiply the number 8 by 2. C. Stop the assessment, and multiply the number 8 by 6. D. Continue to count the patient's breaths for a full 60 seconds

D

For which of the following patients would it be necessary to use a disposable shampoo cap, rather than a shampoo board? A. An older adult woman with a drainage tube in place following a mastectomy B. An older adult man with a history of bleeding problems C. A young woman whose arm and leg have been immobilized on the right side following a car accident D. A young man who has sustained a fracture of the upper spine in a football game

D

For which patient can the nurse delegate to NAP the task of routine blood glucose monitoring? A. Patient with non-insulin-dependent diabetes for whom steroid therapy has been ordered B. Patient with type 2 diabetes who required insulin coverage at the last testing C. Patient with type 1 diabetes who has had nausea and vomiting for 24 hours D. Patient with type 2 diabetes who has had a closed reduction of a fracture of the right wrist.

D

How can the nurse ensure that medication from a single-dose vial is used appropriately? A. Check to see when the medication vial was opened initially B. Write the date and his or her initials on the label when opening the vial C. Draw the entire amount of medication from the vial into the syringe D. Discard the vial and any remaining medication in the vial directly after use

D

How does the nurse minimize the risk of patient infection when preparing medication from an ampule? A. Using a filter needle to draw up the medication B. Preparing the medication in the patient's room C. Applying clean gloves while preparing the medication D. Preserving the sterility of the needle while preparing the medication

D

On the last assessment of a patient's respiration, her respiratory rate was 10 breaths per minute. What should the nurse do when conducting the next assessment of this patient's respiratory rate? A. Count the breaths for 10 seconds and multiply by 6. B. Count breaths for 15 seconds and multiply by 4. C. Count breaths for 30 seconds and multiply by 2. D. Count breaths for 60 seconds

D

The nurse encourages a patient with type 2 diabetes to engage in a regular exercise program primarily to improve the patient's: A. Gastric motility, thereby facilitating glucose digestion. B. Respiratory effort, thereby decreasing activity intolerance. C. Overall cardiac output, thereby resuming resting heart rate D. Use of glucose and fatty acids, thereby decreasing blood glucose level.

D

The nurse has delegated a male patient's perineal care to the NAP. Which statement made by the NAP requires the nurse's follow-up? A. "I will check to see if he cleans himself well." B. "I will let you know if I see any redness or drainage." C. "I will ask him if he is experiencing any pain in that area." D. "I will be sure to use hot, soapy water to be sure he's clean."

D

The nurse has selected a finger as the puncture site to measure the blood glucose level of a female patient with type 2 diabetes mellitus and peripheral vascular disease (PVD). Although all of the actions listed below are appropriate, which one would be of particular benefit to this patient given her medical history? A. Reviewing her current medications B. Inspecting the selected finger for bruising C. Following standard precautions D. Keeping the finger in a dependent position during the puncture

D

The nurse is applying a topical antibiotic and dressing to a burn on the hand of a patient being treated as an outpatient. What is the most important thing the nurse can do to minimize the risk of infection? A. Evaluate the patient's ability to recognize the signs and symptoms of infection B. Perform effective hand hygiene before and after the application C. Instruct the patient not to change the dressing between visits D. Apply the medication using sterile technique

D

The nurse is delegating to NAP the perineal care of a female patient who is totally dependent and confined to bed. Which statement by the NAP requires the nurse's follow-up? A. "I'll ask for assistance if I need help positioning her." B. "I'll see if she's up to the care right now." C. "I'll let you know if I notice any signs of redness or discharge." D. "I'll be sure to use hot, soapy water, since she has been incontinent."

D

The nurse is discussing the risk of falling with the wife of a patient with cognitive impairment. What is the nurse's best response when the patient's wife says, "I don't like him being tied down in the bed?" A. "I'm sure you don't want him to fall again." B. "Can you suggest an alternative?" C. "What did you do to prevent him from falling when he was at home?" D. "We will try all other alternatives before using physical restraints."

D

The nurse is preparing to measure the oxygen saturation level of a patient with obesity. Which action would help ensure an adequate measurement? A. Place the sensor on the ear B. Place the sensor on the bridge of the nose C. Place the sensor on a finger D. Use a disposable tape-on sensor

D

The nurse is preparing to provide perineal care for a female patient who is on bed rest. Which patient position should the nurse use for this care? A. Supine B. Prone C. Side-lying D. Dorsal recumbent

D

The nurse is teaching a patient about ways to reduce blood pressure. What will the nurse include in these instructions? A. Follow your regular healthy diet B. Limit physical activity C. Ensure an adequate daily intake of sodium and fat D. Ensure that your diet has an adequate daily intake of calcium

D

The nurse measures a patient's oxygen saturation level as being 83%. What would the nurse do first? A. Reassess the oxygen saturation in a different location B. Promptly report the assessment data to the charge nurse C. Encourage the patient to rest quietly in bed for 30 minutes D. Ask the patient whether he or she is having trouble breathing

D

The nursing assessment of a 78-year-old woman reveals orthostatic hypotension, weakness on the left side, and fear of falling. On the basis of the patient's data, which one of the following nursing diagnoses indicates an understanding of the assessment findings? A. Activity Intolerance B. Impaired Bed Mobility C. Acute Pain D. Risk for Falls

D

The patient has requested a PRN medication for nausea. Which of the following should the nurse do first? A. Offer dry crackers and ice chips if not contraindicated B. Ask the patient about his allergies C. Explain the specific purpose of the medication D. Check to see when the medication was given last, and make sure the time interval is up.

D

To ensure proper distribution of ear medication after instillation, what will the nurse instruct an adult patient to do? A. Have a family member instill the medication B. Avoid contaminating the medication's applicator tip C. Instill the medication at the time ordered by the provider D. Instill the medication after gently pulling the ear up and back

D

To make sure the drug is delivered properly, what discharge instructions might the nurse give a patient who is being discharged with a dry powder inhaler? A. Rinse your mouth out with water using the inhaler B. Use the inhaler while sitting up in bed C. Keep track of the dosage using the counter on the inhaler D. After inhaling the medication, hold your breath for at least 10 seconds before exhaling

D

Under what circumstances would the nurse assume responsibility for providing denture care for a patient? A. Assessment of the oral cavity shows mucositis due to chemotherapy. B. The patient's previous set of dentures was misplaced or thrown away. C. The dentures belong to the hospital or other facility, rather than to the patient. D. The patient is unable the care for the dentures on his or her own.

D

What can the nurse do to ensure proper site selection for subcutaneous insulin injection? A. Insert the needle at a 30-degree angle B. Select a different anatomical region for each injection C. Ask the patient to relax before inserting the needle D. Systematically rotate sites within the same anatomical location or area.

D

What instruction should the nurse give NAP regarding the appropriate technique when measuring the adult patient's apical pulse? A. Document the patient's pulse rate and rhythm B. Place the patient in the right lateral position before measuring the apical pulse C. Review the patient's previous apical pulse measurements D. Place your stethoscope at the fifth intercostal space over the left midclavicular line

D

What is the best way for the nurse to ensure that the patient does not receive the wrong dose because of a calculation error? A. Ask the pharmacy to calculate the correct dosage B. Consult a current drug book to determine the new dosage C. Defer the calculation process to the provider D. Ask another RN to verify the calculation

D

What is the primary purpose of appropriate hand hygiene? A. To remove all microorganisms from the hands B. To minimize exposure to microorganisms on contaminated sinks, medication containers, catheters, and other hard surfaces C. To leave a protective antimicrobial film on the hands D. To prevent or control the transmission of infectious microorganisms from any source

D

What is the primary reason an unconscious person is placed in a side-lying positioning when mouth care is provided? A. To make the oral cavity easily accessible B. To prevent possible musculoskeletal injury C. To reduce plaque buildup in the mouth D. To reduce the risk of aspiration

D

What is the primary reason for performing care on a male patient with incontinence? A. To provide comfort and a relaxed, refreshed feeling B. To promote personal hygiene while minimizing perineal odor C. To remove all microorganisms from the patient's perineal area D. To reduce the risk of skin breakdown in the patient's genital and perineal area

D

What is the purpose of parting the patient's hair into sections? A. To identify the areas to be groomed B. To style the hair attractively C. To check for pediculosis (head lice) D. To make brushing and combing more effective

D

What will the nurse do after opening a multi-dose vial and withdrawing a dose of medication from it? A. Discard the unused portion of the medication B. Wipe the entire vial with an antiseptic swab C. Send the unused portion back to the pharmacy D. Label the vial with the date it was opened and your initials

D

What will the nurse do right after placing a clean topsheet on the patient? A. Make a cuff with the top of the sheet. B. Make a horizontal toe pleat C. Tuck the remaining portion of the sheet under the foot of the mattress D. Remove the bath blanket

D

What will the nurse instruct NAP to do when measuring a patient's rectal temperature using an electronic thermometer? A. Place the patient in the Fowler's position B. Wear sterile gloves during the process C. Insert the probe in the direction of the knees D. Use the probe with the red tip

D

What will the nurse instruct NAP to do when measuring an adult patient's radial pulse? A. Place the patient in the lateral position before measuring the pulse B. Apply gloves with each patient before measuring the pulse C. Document whether the patient's pulse is bounding or has diminished D. Palpate the patient's inner wrist on the thumb side with the fingertips of your two middle fingers.

D

What would cause the nurse to delay the assessment of a patient's blood pressure? A. Patient is resting in bed, reading a book B. Patient received medication within the last 10 minutes C. Patient is visiting with family D. Patient has just finished having a cigarette

D

When a nurse delegates hygiene care for a male patient to a nurse assistant, the assistant to must use an electric razor to shave the patient with the following diagnosis: A. Congestive heart failure B. Pneumonia C. Arthritis D. Thrombocytopenia

D

When changing the soiled gown of a patient with left-sided paralysis, what will the nurse do first? A. Remove the sleeve from a weaker arm B. Roll the patient into a prone position C. Help the patient assume a side-lying position D. Remove the sleeve from the stronger side

D

When preparing to assist a patient with hair care, why does the nurse first check the patient's scalp for inflammation? A. To determine which type of shampoo to use B. To plan enough time to perform hair care C. To determine if the patient can perform the care independently D. To ensure that the care can be performed without injuring the scalp

D

Which PPE will the nurse wear if there is a risk of a blood splash when caring for a patient? A. Gown B. Gown and gloves C. Gown, gloves, and mask D. Gown, gloves, mask and eye protection

D

Which action by the nurse helps to ensure that the medication is delivered into the muscle when administering an intramuscular injection? A. Using a 1-inch needle B. Inserting the needle at a 45- to 60-degree angle C. Withdrawing the needle immediately after delivering the medication D. Aspirating for blood return before injecting the medication

D

Which action would the nurse carry out first when performing a blood glucose test on a patient with type 1 diabetes mellitus? A. Apply clean gloves to minimize the risk for contamination B. Assess the patient's skin for possible puncture sites C. Ask the patient to wash his or her hands and forearms with warm soapy water D. Determine the patient's preferred puncture site

D

Which example reflects effective documentation of medication administration by a nurse? A. Comparing the written order with the MAR three times B. Providing patient education regarding a medication C. Obtaining a BP before giving a BP medication D. Including the location of an injection site on the MAR

D

Which instruction to nursing assistive personnel (NAP) is most relevant to the proper performance of a fecal occult blood test using a Hemoccult slide? A. "Be sure to wear sterile gloves." B. "Reinforce with the patient the need to use the hat." C. "Is the patient capable of assisting with the collection?" D. "Remember to take samples from two different areas of the specimen."

D

Which nursing action reduces the risk of falling as a patient is getting into or out of the bathtub? A. Add 1 oz of bath oil to the tub water before the patient gets into the tub B. Place an "occupied" sign on the door C. Fill the tube half full of water at 110F to 115F D.Place a skidproof disposable bath mat in front of the tub

D

Which of the following nursing interventions should be implemented to maintain a patent airway in a patient on bed rest? A. Isometric exercises B. Administration of low-dose heparin C. Suctioning every 4 hours D. Use of incentive spirometer every 2 hours while awake

D

Which statement demonstrates an understanding of an importance of communicating changes in the patient's apical pulse rate? A. "The patient's apical pulse is recorded as you asked." B. "The apical pulse is more difficult to hear when the patient is sitting up." C. "The apical pulse is usually slower in the morning than it is in the afternoon." D. "The apical pulse increased from 78 to 110, but the patient had just returned from the bathroom."

D

Which statement made by NAP assigned to care for a patient with dementia requires the nurse to follow up? A. "I encouraged his son and daughter-in-law to stay with him during visiting hours, if possible, even if they run out of things to talk about." B. "He can't see his Foley because it's covered by his boxer shorts." C. "I'll ask the patient every hour or so whether he needs to use the bathroom." D. "He doesn't understand much of what anyone says to him today, so I didn't put in his hearing aids."

D

Which statement might the nurse make to nursing assistive personnel (NAP) assigned to collect a midstream urine specimen from a patient with signs of a urinary tract infection? A. "Obtain 30 to 60 mL of midstream urine." B. "The urine has a foul odor." C. "Teach the patient to collect the urine specimen." D. "Be sure to maintain aseptic technique."

D

Which statement or question best illustrates the nurse's understanding of the role of NAP in the instillation of ear medication? A. "Did you let the ear medication warm to room temperature?" B. "Do you think the patient is capable of instilling her own ear drops?" C. "Please tell the patient that the medication may make him dizzy when he stands up." D. "Be sure to keep the patient on her side for a few minutes, because I administered her ear drops."

D

Which statement or question best illustrates the nurse's understanding of the role of NAP in the instillation of eye medication? A. "Did you let the eye medication warm to room temperature?" B. "Do you think the patient is capable of instilling his own eye drops?" C. "Be sure to slightly hyperextend her neck when instilling the medication." D. "Her vision may be temporarily impaired, so please help her to the bathroom."

D

Why would the nurse avoid placing nitroglycerin ointment over a scar on an otherwise suitable area of the upper arm? A. The ointment will stick to the scar tissue B. The ointment is likely to irritate the scar tissue C. The ointment may cause the scar to become hypertrophic D. The scar tissue may interfere with absorption

D

You have assigned a new NAP to take routine vital signs. An experienced NAP has been asked to retake a blood pressure that the newly hired NAP has taken three times this week. As the nurse, what action do you take? A. Do not delegate vital signs to the NAP B. Delegate only temperature and respiratory rate to the NAP C. Report the NAP to your supervisor D. Observe the NAP as he or she obtains a blood pressure and pulse on a patient

D

Your assigned patient has a leg ulcer that has a dressing on it. During your assessment you find that the dressing is saturated with purulent drainage. Which action would be best on your part? A. Reinforce dressing with a clean, dry dressing and call the health care provider B. Remove wet dressing apply new dressing using sterile procedure C. Put on gloves before removing the old dressing; then obtain a wound culture D. Remove saturated dressing with gloves, remove gloves, then perform hand hygiene and apply new gloves before putting on a clean dressing

D

The effluent from an ileostomy is: Type 1 like pebbles Clear in color Solid Different from that of a colostomy

Different from that of a colostomy

__________________ describes the tendency of cells and tissue to reduce in size and function in response to prolonged inactivity resulting from bed rest, trauma, casting of a body part, or local nerve damage.

Disuse atrophy

A patient's gastric residual volume was 250 ml at 0800 and 350 ml at 0900. What is the appropriate nursing action? Do no reinstall aspirate and hold the feeding until you talk to the primary care provider Raise the head of the bed to at least 45 degrees Assess bowel sounds Position the patient on his or her tight side to promote stomach emptying

Do no reinstall aspirate and hold the feeding until you talk to the primary care provider

What are some signs and symptoms that may put a patient at risk for malnutrition? Flatulenced Shortness of breath Enteral feedings Dysphagiac

Dysphagiac

A colostomy pouch should be changed every Every 3 to 7 days Other day Once a month Every day

Every 3 to 7 days

You are transferring a patient from the bed to the chair. In order to prevent abnormal twisting of the spine you should: Face the direction of movement Align your body towards the chair and away from the patient before you begin to lift. Maintain a narrow base of support to allow for greater stability Avoid using antigravity muscles while lifting

Face the direction of movement

The provider has ordered for you to teach Mr. Jones ROM exercises to the left elbow. Describe how you would instruct him to perform flexion and extension ROM exercises.

Flexion: Go ahead and bend your elbow forward so that your arm moves toward the shoulder joint and your hand becomes level with your shoulder. Extension: Lower your hand to be able to straighten your arm

In what position, should the patient be for the insertion of a nasogastric tube? High fowlers At 30 degrees Lateral Supine

High fowlers

Mr. Jones is an 80-year-old male, admitted for a right total hip arthroplasty. List three nursing interventions that you will initiate with Mr. Jones to prevent respiratory complications due to immobility.

I would promote deep-breathing exercises, incentive spirometry, and controlled coughing to prevent respiratory complications.

A patient with fecal incontinence is at risk for what? Allergies Decrease water intake Constipation Impaired skin integrity

Impaired Skin integrity

In order to maintain proper body alignment and posture you must: (Select all that apply) Ensure the bed is at its lowest to prevent the risk of falls Increase balance by bringing the center of gravity closer to the base of support Widen the base of support by separating the feet to a comfortable distance Bend at the hips and lift using the muscles in your back for added support

Increase balance by bringing the center of gravity closer to the base of support Widen the base of support by separating the feet to a comfortable distance

Select all that apply: Loss of walking independence causes what? -Leads to less injuries -Increases hospital stays -Increases the need for rehabilitation services or nursing home placement -Promotes quicker recovery

Increases hospital stays Increases the need for rehabilitation services or nursing home placement

You are assisting a toddler during ambulation. You notice that as he walks, his legs and feet are far apart and slightly everted. You recognize that this: May be corrected with isometric exercises Is a sign of likely a result of underdeveloped muscle tone, requiring aggressive physical therapy Is normal in toddlers Is usually seen in older adults

Is normal in toddlers

Mrs. Doe a confused patient has a nasogastric (NGT) tube and is receiving enteral nutrition, you notice that she starts to have respiratory distress. What could be the reason for respiratory distress? Electrolyte imbalance NGT tube displacement Dehydration Occlusion of NGT tube

NGT tube displacement

When assisting a patient with standing following a prolonged period of bed rest, what physical response should the nurse be most prepared to assess? Steadiness of gait Dependent edema Respiratory rate Orthostatic hypotension

Orthostatic hypotension

When performing a guiac fecal occult blood test, the stool samples are: Placed on all over the guiac (card) Taken from the same area of the stool sample Placed in only one small box of the guiac (card) paper Placed in both small boxes of the guiac (card) test paper

Placed in both small boxes of the guiac (card) test paper

The nurse sees the nursing assistive personnel perform the following for a patient receiving continuous enteral tube feedings. What intervention does the nurse need to address immediately? Fastens the tube to the gown with tape : elevates the head of the bed 45° Places the patient supine while giving a bath Fastens the tube to the gown with tape Performs oral are for the patient

Places the patient supine while giving a bath

In what part of the digestive system is where nutrients are absorbed? Stomach Colon Pancreas Small intestine

Small intestine

During the administration of an enema a patient reports abdominal cramping, what do you need to do? Stop the installation Ask the patient to take deep breaths to decrease the pain Add soap suds to the enema Tell the patient to bear down Ask the patient to take deep breaths to decrease the pain

Stop the installation

You are teaching your patient with decreased mobility to the right leg how to use a cane. Which action indicates proper cane use by the patient? The patient keeps the cane on the right side of the body The patient slightly leans to one side while walking The patient keeps two points of support on the floor at all times. After the patient places the cane forward, he or she then moves the left leg forward to the cane

The patient keeps two points of support on the floor at all times.

Straining while having a bowel movement indicates that; The patient needs to pass gas The patient need to eat more nutrients The patient needs to eat less The patient may need changes in diet or fluid intake

The patient may need changes in diet or fluid intake

Pouches should be emptied when they are one-third or half full because of what complication? The weight of the pouch may disrupt the seal and cause leaks No complication will happen Rupture of bag The need for another ostom

The weight of the pouch may disrupt the seal and cause leaks

The individual of average weight and height without a chronic illness on bed rest loses muscle strength from baseline levels at a rate of _____ % a day.

Three

What is the most important reason to encourage the use of antiembolic stockings? To facilitate: Venous return Arterial insufficiency Early ambulation Skin integrity

Venous return

What is the gold standard to confirm NGT tube placement? Oxygen saturation X-ray PH Gastric residuals

X-ray

48. The nurse is inserting a small-bore nasoenteric tube before starting enteral feedings. What is the correct order of steps to perform this procedure? a. 1. Place patient in high-Fowler's position. b. 2. Have patient flex head toward chest. c. 3. Assess patient's gag reflex. d. 4. Determine length of the tube to be inserted. e. 5. Obtain radiological confirmation of tube placement. f. 6. Check pH of gastric aspirate for verifying placement. g. 7. Identify patient with two identifiers. i. 7, 1, 3, 4, 2, 5, 6 ii. 1, 3, 4, 7, 2, 6, 5 iii. 7, 1, 3, 2, 4, 6, 5 iv. 1, 7, 3, 2, 4, 5, 6

i

40. What is the correct sequence for suctioning a patient? a. 1. Open kit and basin. b. 2. Apply gloves. c. 3. Lubricate catheter. d. 4. Verify functioning of suction device and pressure e. 5. Connect suctioning tubing to suction catheter. f. 6. Increase supplemental oxygen. g. 7. Reapply oxygen. h. 8. Suction airway i. 6, 4, 3, 1, 2, 5, 8, 7 ii. 4, 6, 1, 2, 3, 8, 5, 7 iii. 4, 6, 1, 3, 2, 5, 8, 7 iv. 6, 4, 1, 3, 2, 5, 7, 8

iii

8. What is the correct order in which elastic stockings should be applied? a. Identify patient using two identifiers. b. Smooth any creases or wrinkles. c. Slide the remainder of the stocking over the patient's heel and up the leg d. Turn the stocking inside out until heel is reached. e. Assess the condition of the patient's skin and circulation of the legs. f. Place toes into foot of the stocking. g. Use tape measure to measure patient's legs to determine proper stocking size. i. 1, 5, 7, 4, 6, 2, 3 ii. 1, 7, 5, 4, 6, 2, 3 iii. 1, 5, 7, 4, 6, 3, 2 iv. 1, 5, 4, 7, 6, 3, 2

iii

69. Match the following steps for administering a prepackaged enema with the correct order in which they occur. a. 1. Insert enema tip gently in the rectum. b. 2. Help patient to bathroom when he or she feels urge to defecate. c. 3. Position patient on side. d. 4. Perform hand hygiene and apply clean gloves. e. 5. Squeeze contents of container into rectum. f. 6. Explain procedure to the patient. i. 6, 3, 4, 1, 5, 2 ii. 6, 4, 1, 3, 2, 5 iii. 4, 6, 3, 1, 2, 5 iv. 6, 4, 3, 1, 5, 2

iv

What population is most affected developmentally by prolonged bed rest? infants children adults older adults

older adults

Immobilized patients with a history of ___________ osteoporosis are at high risk for accelerated bone loss.

primary


Conjuntos de estudio relacionados

U.S. Government: Chapter Three Review and Study Guide

View Set

Quiz 2: U.S. States, Capital Cities, and the Global Traveler

View Set

Factors that Affect Biodiversity

View Set